Problemas Resueltos de Magnetismo

Problemas resueltos de Electricidad y Magnetismo E.T.S.I.T. Universidad de Las Palmas de Gran Canaria ELECTRICIDAD Y M

Views 157 Downloads 1 File size 4MB

Report DMCA / Copyright

DOWNLOAD FILE

Recommend stories

Citation preview

Problemas resueltos de Electricidad y Magnetismo

E.T.S.I.T. Universidad de Las Palmas de Gran Canaria

ELECTRICIDAD Y MAGNETISMO. Electrostática-Vacío

1) Suponiendo una nube de electrones confinada en una región entre dos esferas de radios 2 cm y 5 cm, tiene una densidad de carga en volumen expresada en coordenadas esféricas:

ρ = − 3⋅10−8 ⋅ cos2φ (C ⋅ m−3) v

R4

Calcular la carga total contenida en dicha región. 2) Sobre dos placas paralelas e indefinidas, separadas por una distancia d, se distribuyen respectivamente las densidades de carga superficiales: ρs,1=2 Cm-2, ρs,2=4 Cm-2 . Calcular el campo entre los dos planos y en el espacio a derecha e izquierda de los mismos.

3)

Y r φ O

X

Sobre la semicircunferencia indicada en la figura se distribuye una densidad de carga lineal ρl=ρo cos φ. a) Calcular la carga total distribuida sobre la semicircunferencia. b) Calcular el campo en el punto O.

Z θ

4) Sobre una capa semiesférica de radio R, tenemos una distribución superficial de carga uniforme ρs=1 Cm-2. a) Calcular la carga total en la capa semiesférica. b) Calcular el campo eléctrico en el centro O de la figura. Y

R O

X

5) En el centro de una placa de espesor d e indefinida en las otras dos direcciones, existe un hueco esférico de radio a. En la placa, excepto el hueco, se distribuye una densidad de carga uniforme ρv. Calcular el campo en el punto A, a una distancia d/2 de la placa.

A

a d/2 d

ELECTRICIDAD Y MAGNETISMO. Electrostática-Vacío

6) Tenemos un cilindro indefinido de radio a, sobre él se distribuye una densidad de carga en coordenadas cilíndricas ρv=ρo sen(πr/a), siendo ρv=0 para r>a. a) Calcular el campo eléctrico. b) Si situamos una carga negativa sobre el eje del cilindro, ¿será estable la situación de equilibrio de dicha carga?.

2b 7) Una esfera se taladra diametralmente, dejando un hueco cilíndrico de radio b=10-2⋅a. El hueco se puede considerar filiforme en comparación con el radio a de la esfera. En la esfera, salvo en el hueco cilíndrico, se distribuye una densidad de carga uniforme ρv. Aplicando el principio de superposición, calcular el campo eléctrico E en el punto P.

2a a O

P

8) Calcular y dibujar el campo y el potencial, E y V, en función de R para la distribución esférica de carga:

ρ=

1/2

 ρo(0R / a)

parapara Ra / a

v



9) Sobre un plano indefinido tenemos dos distribuciones de carga. Una densidad superficial de carga uniforme -ρs sobre un círculo de radio R y otra de signo contrario ρs sobre el resto. Aplicando el principio de superposición, calcular el campo eléctrico sobre el eje perpendicular al círculo y que pasa por su centro. 10) Sobre un disco plano de radio R se distribuye una carga superficial que varía radialmente de la forma:



  r 2 si r < R

ρo   ρs =



 

0R si r > R

siendo r la distancia al centro del disco. Calcular el potencial y el campo en el eje perpendicular al disco y que pasa por su centro.

ElectrostÆtica

PROBLEMAS DE ELECTROST`TICA - VAC˝O

 Ejercicio

1

La carga total vendrÆ dada por:

ρϑ = dqdv ⇒ Q = ∫v ρϑdv

con dv =

R2sen( )θ dRdφdθ.

Sustituyendo la densidad de carga en volumen, e integrado en el volumen especificado:

Q

 Ejercicio

C

2

ElectrostÆtica

Figura 2.1

Figura 2.2

Para resolver el problema, dividiremos el espacio en tres zonas: 1) Zona comprendida entre los planos. 2) Zona a la derecha, y > d. 3) Zona a la izquierda, y < 0. Para una sola lÆmina, por simetr a (por ser infinita), el campo elØctrico E es perpendicular a ella y tiene la misma magnitud en ambos lados. La aplicaci n del teorema de Gauss en el cilindro de la figura 2, colocÆndolo de forma que sea cortado por la lÆmina con las tapas paralelas a su superficie, se obtiene la relaci n siguiente:

→E Siendo ds la superficie en las tapas. Aplicando lo sabido para una lÆmina a las del problema, distinguimos tres regiones distintas: 1) Zona comprendida entre los planos. En esta zona el campo total serÆ la suma de los campos debidos a cada una de las distribuciones, teniendo en cuenta que tienen la misma direcci n pero sentidos opuestos. E= E1 + E2 Sabemos que:

Sustituyendo en la ecuaci n anterior:

ElectrostÆtica

Sustituyendo los valores de σ1 y σ2 :  2 4 u E = − (u y ) = − y 2 ⋅ε 0 ε0 2) Zona a la derecha de los planos, y > d. Se procede de forma similar al apartado anterior, con la condici n particular de que en esta zona los campos creados por las dos distribuciones tienen la misma direcci n y sentido, es decir, los dos tienen sentido hacia y > 0. Sabemos que:

Sustituyendo en la ecuaci n anterior:

Sustituyendo los valores de σ1 y σ2 :  2 4 3 u E = + (u y ) = ⋅ y 2 ⋅ε 0 ε0 3) Zona a la izquierda de los planos, y < 0. Calculamos el campo de manera similar a los casos anteriores, pero ahora los campos tienen sentido hacia y < 0, por tanto:

Sustituyendo en la ecuaci n anterior:

Sustituyendo los valores de σ1 y σ2 :  3 u 2 4 E = − − (u y ) = − ⋅ y 2 ⋅ε 0 ε0  Ejercicio 3 1) Para calcular la carga total distribuida sobre la semicircunferencia, teniendo en cuenta la definici n de la densidad lineal de carga:

ElectrostÆtica

ζ1 = lim∆l→0 ∆q ∆l Con lo que la carga total distribuida es: Q = ∫ 1ζ1dl Donde el dl para el problema en concreto es: dl = r ⋅ dφ y la densidad lineal de carga es:

ζ1 =ζ0 cosφ Q

=



ζ

1 0

cosφ⋅rdφ

EstÆ claro que la integral ha de ser evaluada entre -π/2 y π/2:

Q r y ζo pueden salir de la integral al no ser dependientes de φ quedando: Q r π2 π2 =ζ 0 r − cosφdφ =ζ 0 r (senφ )−π = 2ζ 0

∫π

2

2

2) El campo en el punto O, en una distribuci n lineal de carga, se calcula como:

E( )r’ =  4πε1 0 ∫ζ1(rr’−’−rr3)dl

Teniendo en cuenta que -r=0 - r = r (cosΦ ax + senΦ ay) - dl = r •dΦ - ζl = ζo cosΦ - Evaluando la integral entre -π/2 y π/2: E(0) =

1 π2 ζ0 cosφ− r(cosφax3+ senφay )rdr 4πε0 −π∫ r 2

Ahora los tØrminos no dependientes con Φ se pueden sacar fuera de la integral, tanto las r como ζo, quedando lo siguiente:

ElectrostÆtica E r2 (0) = − ζ 0 3 4πr ε 0

π2

cosφ(cosφa ∫ π

x

+ senφa y )dφ

− 2

EstÆ claro que por la simetr a que presenta el problema las componentes en el eje y del campo se van a ir anulando unas con otras, por ello la integral que queda para el eje y se ha de anular: E (0) = − ζ 0 4πr ε 0

π2

π2

−π 2

−π 2

2 ∫cos φa x dφ + ∫cosφsenφa y dφ

Para la primera integral se opera con el Ængulo doble:



π2

∫cos φdφa π 2

2

π2

=

∫ π 2

x

π2



ax =

1 + cos 2φ φ sen2φ  =2+ 4  2 

−π 2

π4 + π4 ax = π 2 ax La segunda es inmediata y como se hab a dicho ha de anularse:

ay = 0ay Quedando la siguiente expresi n para el campo:

E(0) = − 4πζr0ε0 ⋅π2 az + 0ay  La segunda parte del problema tambiØn se podr a haber hecho con la expresi n que se dio en clase, la cual estÆ un poco mÆs simplificada:

E( )r’ =  4πε1 0 ∫ζr1dl2 ’ aR

Para utilizarla se ha de tener en cuenta la direcci n y sentido del vector aR

ElectrostÆtica

 Ejercicio

4

1) La carga total en la capa semiesfØrica. Sabemos que, para una densidad superficial de carga: Qs = ∫sρs ⋅ ds Para una semiesfera: Qs Qs Q s = 2⋅π⋅ R 2 2) El campo elØctrico en el centro O de la figura.

Como podemos observar, la componente horizontal del campo se anula, quedando s lo la vertical. dq = ρ⋅ds

dE 

Calculamos el campo infinitesimal efectivo dEef = −dE ⋅cosθ⋅dθ(a z ):

ElectrostÆtica 

π

E = 4πε1⋅[ ]φ 02π ⋅sen22θ0 2 (az )    E = − 1 4 ⋅ (a z ) ε  Ejercicio 5



• d/2 A

E 

  E A = 1A +E2 A

↓ ∞ < - - - - - - - - - -- - - - - - -> d ∞

 1A

ElectrostÆtica E

donde E1A y E2A son, respectivamente, los campos en A debidos a una placa maciza con densidad de carga ρv y a una esfera a centrada con densidad ρv. Para calcular E1A, al ser una placa indefinida podemos aplicar GAUSS. E1A  aplicando el teorema de gauss:

∫ E.dS= Qvε

o

 dS 

 dS

 E1 A

 E1 A dS

-------------------- d

E1A = ∫ E.dS perpendicular a



+







E .dS



como en la superficie lateral dS

es

ElectrostÆtica E1A entonces

tapas

integral se anula.

E1A •

lateral

dS = E ⋅dS cos90° = 0 la



E1A = ∫E.ds.Cos0° = E1 ∫ dS = E ∫dS = E.2(∆S)= 2E∆S tapas

tapas

Q

2E S1.∆= ε

2E1A.∆S =

v

o

tapas

- - - - > Qv = V∫ρv..dV = ρvV∫dV = d.∆S

ρv. . d S ∆ ε o

ρ

.d

- - - - > E1A = 2εv 

o

E1A = ρ2εv.do ax E2A  En una distribuci n de carga con simetr a esfØrica, el campo creado en el exterior es equivalente al creado por una carga puntual en el centro, de valor la carga encerrada por la distribuci n.

.A

- ρv •A

•A

 E2 A

E2a

d/2 ----- QV = ∫ −

ρ V

.dV = −

V

ρ V

∫dV

V

4

QV = −ρv. π.r3 3 

por la ley de Coulomb:



1

E = .QV2 4πεO d −ρV .4π.r3

E2A =4π.ε3O.d 2 = 3−.ερov..da23 = 3−.ερov..da23 .. ax

por el principio de superposici n:

ElectrostÆtica

EA = ρ2Vε.od − 3ρεvo..ad32 •ax

EA = ερOV . d2 − 3.ad32  . ax

 Ejercicio

6

1) Calcularemos el campo elØctrico mediante Gauss. Para ello hemos de considerar dos casos: a) Caso de superficie gaussiana con a > r

Aplicamos Gauss considerando la superficie gaussiana un cilindro interior de

radio r:

∫Eds= ∫ s

tapas

Eds+ ∫lateralEds⇒E∫sds = E⋅2πrL

ElectrostÆtica El campo es radial. En las tapas,



por lo que se anula. En la superficie natural ds



es paralelo a dEs con lo que se anula el carÆcter vectorial y es perpendicular a E consideramos a E como una constante. Tomando L = 1 y aplicando Gauss nos quedamos con la expresi n: Q E ⋅ 2πr = v ε Ahora buscamos Qv, la carga libre encerrada por la superficie gaussiana, integrando ρv:

Qv = ∫v ρvdv = ρ0 ∫or sen  πar rdr 2∫0πdφ∫10 dz = 2πρ0 ∫0r sen πar rdr = 2πρ0I

Haciendo la integral I por Partes:

u=r dv=senπrdr a

du= dr v = − aCosπr πa

Sustituyendo en la integral: r

Qv =− rπa Cosπar 0 −∫0r − πa cosπar dr 2

Qv = − πa r cos πar  + πa  sen πar 

Qv = πa πa senπar − rcosπar  = I

Por lo que nos queda: Qv = 2πρπ0a ⋅ πa senπar  − r cosπar 

ElectrostÆtica

Que sustituyendo en la expresi n de Gauss nos da el campo:  E = πρrε0a πa senπar  − r cosπar •(ar ) o b) Caso para el que la superficie gaussiana tiene de radio a > r:

Por un procedimiento anÆlogo al anterior llegamos a la expresi n: E ⋅ 2πr =

Qv

ε

Ahora la superficie Gaussiana es exterior al cilindro, por lo que tenemos que tener en cuenta que existirÆn dos Qv diferentes en r > a y r ≤ a.

Qv(r > a) ⇒ ∫vρvdv = 0 Qv(r ≤a)⇒∫vρvdv =Qv anterior con r = a En la expresi n de Qv del apartado (a), sustituimos r por a, con lo que obtenemos la siguiente Qv y consecuentemente tambiØn el campo.  2 Qv = 2ρ0a ⇒ E = a 2ρ0 (ar) ε0πr 2) Introducimos una carga negativa (-q) en el eje del cilindro. Para que la carga se encuentre en equilibrio no debe existir ninguna fuerza actuando sobre ella. Para ello usamos el campo existente en el eje (r = 0):

ElectrostÆtica

F= (−q)•E(r=0) El campo en el eje es una indeterminaci n del tipo 0/0, por lo que para calcularlo tomamos el l mite cuando r tiende a 0 en la expresi n del campo que obtuvimos en el primer apartado: Lim→ 

aρ0 

a

π



π r

sen  a



−

 π r

r cos  a

 =0

0 = INDETERMINACION r 0 πε0 r 

 

Aplicamos L Hopital, y derivando arriba y abajo (derivando respecto a r):

Limr→0 πεaρ0    πa π a cosπar  − cosπar  + r π a sen πar  0

E=

aρ0

[1−1+ 0]

=0

πε0 Al ser el campo 0, la fuerza tambiØn serÆ 0, por lo tanto podemos deducir que la carga estÆ en equilibrio. Ahora, para saber si el campo es estable, averiguamos el sentido de la fuerza que existe en las proximidades del eje. Si estas fuerzas hacen que la carga tienda hacia el eje, se encuentra en estabilidad. Si por el contrario las fuerzas hacen que la carga tienda hacia el exterior, el equilibrio en el eje serÆ inestable. La carga en el interior del cilindro es siempre positiva:

r ∈[0,a]⇒ a ∈[0,π]⇒ρv > 0, ∀r π r

Por lo que el sentido del campo elØctrico es hacia el exterior del cilindro. Al ser la carga puntual colocada en el eje negativa, la fuerza que experimentar a si se separara del eje ir a en contra del campo, es decir, hacia el interior del cilindro. Por lo que la situaci n es de equilibrio estable.

ElectrostÆtica Ejercicio 7 Para calcular el campo en el punto P aplicaremos el principio de superposici n, calculando el campo creado por la distribuci n de carga en toda la esfera, con una densidad volumØtrica ρv, y luego el campo creado por el cilindro si Øste estuviera cargado por una densidad volumØtrica -ρv. As , sumando ambos, habremos calculado el campo creado por la esfera taladrada, puesto que la densidad volumØtrica de carga en el hueco cil ndrico es nula. Campo creado por la esfera Aprovechando la simetr a del problema, utilizaremos el teorema de Gauss para calcularlo. La superficie gaussiana serÆ esfØrica y de radio OP. OP = (2a)2 − a 2 = 3a

∫ E.dS= Q /ε v

0

E.4π( 3.a) 2 = Qvε0 Qv

El campo en el punto P, serÆ pues: E( p) = aρv /9ε0 ay Campo creado por el cilindro (densidad volumØtrica -ρv) Al ser b a.

Operamos de forma similar al apartado anterior, pero en este caso los l mites de integraci n para la distancia en la carga son a/ 2 y a (hay que considerar la carga total que existe en la distribuci n).

Operando:

E = 2ρ0 2 ⋅a31− 18 ⋅ r12 (ur ) 7ε0  1.2) CÆlculo el potencial. a)

r > a.

Calculamos el potencial entre cero e infinito, teniendo en cuenta que el potencial en el infinito es igual a cero:

Vc  r12 dr

a

Operando:

2ρ0

Vc =



⋅a3 1−  18 

1

2 ⋅ r 7ε0



ElectrostÆtica

b) a/ 2 ≤ r ≤ a. Operamos de igual modo que en el caso anterior pero hemos de tener la continuidad del potencial, o sea que Vb (a) =Vc (a) .

r

( ) dr

2ρ012 ⋅ 12 r 72 − a 2

72

Vb (r) −Vc (a) = −∫ 7ε0ar a

r

( ) dr

Vb (r) = Vc (a) − ∫a 7ε2ρ0a012 ⋅ r12 r 72 − a 2

72

Operando:

(

V rb ( ) = V ac ( ) −2ρ012  2 ⋅ r 52 − a 52   a2 72 7ε0a 5

) −   1 1 

⋅ a − r 

c) r < a /2.

En esta zona, no existe campo elØctrico, por lo que el potencial a de ser constante e igual al potencial en a/2.

( )

V a (r) =V b a2 Ejercicio 9

El campo elØctrico cumple el principio de superposici n, de forma que podemos calcular el campo como la suma del creado por un plano infinito de densidad superficial uniforme ρs, mÆs el creado por un disco en el lugar del c rculo de radio R, con densidad -2ρs.

     E=Eplano +Ecirculo =Ez=0 +E−2ρ

ElectrostÆtica 

• Campo creado por el plano infinito (

E

z=0

):

Podemos hallarlo a partir del Teorema de Gauss, al ser el plano infinito.

S1 ∪S2 ∪S3 =Sgauss



Usando una superficie de Gauss cil ndrica que atravesase al plano perpendicularmente conseguimos varias simplificaciones muy interesantes:

0 1) ∫S3E ⋅ dS3 =  El campo es perpendicular al plano y dS3 (en la superficie

2) lateral) es paralelo a Øste. Por tanto, el producto escalar del integrando es nulo. ∫ E⋅ dS= ∫ E⋅ dS2  El flujo del campo en la superficie gaussiana queda:: S1

1

S2

2∫S1E⋅ dS1 = Q 3)

2

∫S2E⋅ dS2 = Qε0

 Ya que el Ængulo que forman

E

S

yd

1

es 4) siempre 0, son paralelos y, por tanto el coseno vale 1. 2∫S1E ⋅ dS1 = 2 ⋅ E∫S1dS1 (El campo es constante en la superficie S1). La carga contenida en la superficie gaussiana es: Q = ρs ⋅ S1 . Con lo que obtenemos que el campo creado por un plano infinito es:

ElectrostÆtica

E=



z



ρs E plano 2 ⋅ ε0

 s ρ a =2 ⋅ ⋅ε 0

• Campo creado por el disco de radio R de densidad -2ρs ( E−2ρ): Para este caso, no podemos usar el Teorema de Gauss y, por tanto, aplicaremos la f rmula del potencial primero y luego, a partir de Øste, hallaremos el campo elØctrico. r



r

' = z2 + r2  Distancia de

un punto del c rculo al punto campo.

Por definici n, el potencial elØctrico en un punto del eje que pasa por el centro de un c rculo de densidad -2ρs es:

ρ

V( z ) = 4 0 ∫ 10 2π R∫ 2 0−2 s ⋅r⋅dr⋅d z 2 + rϕ πε

V(z)

= − ρs

ε



R

0

V( z )

πρ

= 4−4 πε0

∫ r⋅z s

0

R

dr 2

+r

2



zr2 dr+ r2  La integral sale haciendo un cambio de

variable: 0

;

a = z2 + r2 dadr = 2 ⋅ r



V(z) =

−ρs da 

2ε 0



a

V( z ) = − ρs ε0 V(z) = −ερ0s

[ z +r ] 2

2

0R



 E−2ρ = − ∂∂Vz = ερ0s ⋅ R2z+ z2 −1⋅az

(z

2

2

+R −z

)

ElectrostÆtica 

Luego, el campo finalmente será:    E = E plano + E−2ρ = ρ s 2ε 0

2z   1 ⋅ − + ⋅a z R2 + z2   

Ejercicio 10

La formula que define el potencial es la siguiente: V z(

)=∫

ρ

d ds s

Teniendo en cuenta que d es la distancia al punto del eje en que calculamos el potencial. En el disco:  r

2

ρ ρs =

  R

0

ds = rd drϕ d = (z2 + r2) Quedando la integral:

Y resolviendo queda: V z( ) = ρ00 2 13(z2 + r2)32 − z z2( 2 + R2)12 + 23z3  2εR  Ahora calcularemos el campo por la formula del gradiente, sabiendo de antemano que, por la simetr a del problema, s lo vamos a tener campo en le eje z :

ElectrostÆtica E = − ∂V uz = 2ερ00R2 z z( 2 + R2)12 − z z3( 2 + R2)−21 −2z u2 z ∂z

ELECTRICIDAD Y MAGNETISMO. Electrostática-Medios materiales.

1) Una carga puntual positiva Q está en el centro de una capa conductora esférica con radio interior R i y radio exterior Ro. Determine E y V como funciones de la distancia radial R. 2) Suponga un tubo de cobre muy largo con radio exterior de 3 cm y radio interior de 2 cm, que rodea una línea de carga de 60 pCm-1 situada en su eje. Calcular: a) E en r=1 m, 2.5 cm y 1.5 cm. b) La diferencia de potencial entre la superficie interior y la exterior del tubo. 3) Considere dos conductores esféricos con radios b 1 y b2 (b2>b1), conectados por un alambre conductor. Se deposita una carga total Q en las esferas. La distancia entre los conductores es muy grande en comparación con los radios de las esferas, de modo que las cargas en los conductores esféricos se distribuyen uniformemente. Calcular las densidades de carga superficial y las intensidades de campo eléctrico en la superficie de las esferas. 4) Un cilindro conductor de radio R y longitud L, lleva una carga Q. Coaxialmente con él se disponen dos coronas cilíndricas conductoras. La primera, de radios R 1 y R2, lleva la carga Q´, y la segunda, de radios R 3 y R4, está conectada a tierra. Calcular: a) la distribución de cargas y sus respectivas densidades. b) el campo eléctrico en las distintas regiones del espacio (suponer los cilindros muy largos). c) el potencial eléctrico en las distintas regiones del espacio. 5) Sea un conductor, en el que existe una cavidad interior, sometido a un campo eléctrico. Hallar el campo eléctrico existente en el interior de la cavidad así como la densidad de carga en la superficie de ésta. 6) Expresar la energía almacenada por varios conductores independientes entre sí. 7) Una esfera conductora de radio R1 y carga Q, se rodea de una corona esférica conductora concéntrica de radios R2 y R3, siendo R2R1, que se llena con un dieléctrico perfecto de permitividad relativa εr=a/R, en la que a es una constante y R la distancia al centro del condensador. 11) Calcular para una carga puntual en el centro de una esfera dieléctrica el vector de polarización y las densidades de cargas ligadas. Dibujar D, E y V en función de r. Emplear Q=10-9 C, R=2 cm, εr=3. Repetir estas gráficas en ausencia de la esfera dieléctrica. 12) Una esfera dieléctrica de radio a está polarizada de forma que P=(K/R)ar, siendo ar el vector unitario radial. a) Calcular las densidades volumétrica y superficial de carga ligada. b) Calcular la densidad volumétrica de carga libre. c) Calcular el potencial dentro y fuera de la esfera. d) Representar gráficamente la variación del potencial con la distancia.





13) Una esfera de dieléctrico simple está uniformemente polarizada en la dirección del eje z, con

P = 2·10−6

az (Cm-2). Calcular: a) las densidades de carga de polarización. b) el potencial eléctrico en el centro de la esfera. c) demostrar que la densidad de carga libre en el dieléctrico es nula.

14) En un material, de constante dieléctrica ε, existe un campo eléctrico uniforme E. Si se practica una cavidad esférica en el interior del material, calcular el valor del campo eléctrico existente en el centro de la cavidad. 15) Dos medios dieléctricos con permitividades ε1 y ε2 están separados por una frontera libre de cargas. La intensidad de campo eléctrico en la interface en el medio 1 tiene magnitud E 1 y forma un ángulo α1 con la normal. Determine la magnitud y la dirección de la intensidad de campo eléctrico en dicho punto de la interface en el medio 2. 16) Sea un condensador de placas plano-paralelas rectangulares. La superficie de cada placa es S, y están separadas una distancia l. Despreciando los efectos de borde, si se aplica una tensión constante V o entre las placas calcular: a) El campo eléctrico en el interior, la densidad de carga superficial en las placas, la energía almacenada por el condensador y su capacidad. b) Repetir el apartado a), suponiendo que se introduce un dieléctrico de dimensiones l/2 x S, y permitividad relativa εr. c) Repetir el apartado b), pero suponiendo que se desconecta la fuente de tensión antes de introducir el dieléctrico.

ELECTRICIDAD Y MAGNETISMO. Electrostática-Medios materiales. 17) Disponemos de dos condensadores idénticos, de placas plano-paralelas, cuya superficie es S y espesor d, como indica la figura. Entre las placas existe un dieléctrico de permitividad ε = 100εo. Un vez cargados con un diferencia de potencial Vo, y desconectada la batería, en un instante dado se fractura el dieléctrico entre las placas del condensador (1), de forma que se abre una fisura plana y

(2 )

(1 ) 0.01·

d S

paralela a las placas, de espesor 0.01·d. Calcular: a) los vectores Ey Den los condensadores (1) y (2) antes y después de la fractura. b) la diferencia de potencial entre las placas de los condensadores tras la fractura.

18) Cuando se usa un cable coaxial para transmitir energía eléctrica, el radio conductor interior está determinado por la corriente de carga, y el tamaño total por la tensión y el tipo de material aislante que se utilice. Suponga que el radio del conductor interno es ri= 2 mm, y que el material aislante es poliestireno, cuya constante dieléctrica relativa y rigidez dieléctrica son, respectivamente, 2.6 y 20·10 6 V/m. Determine el radio interior, ro, del conductor externo para que, con una tensión aplicada entre los conductores externo e interno de 10 kV, la intensidad máxima del campo eléctrico en el material aislante no exceda el 25% de su rigidez dieléctrica. 19) Un condensador de placas plano-paralelas, separadas una distancia d, tiene un dieléctrico en su interior, ausente de cargas libres, cuya permitividad dieléctrica relativa, εr, depende de la distancia a una de las placas, x. Calcular la capacidad del condensador si εr viene dada por:

ε =r

1 x2 1− 3d2

20) Dentro de un condensador de placas plano-paralelas, de sección A y espesor d, introducimos un dieléctrico de permitividad no uniforme, siendo y la dirección perpendicular a las placas. Despreciando los efectos de borde y en caso de no existir cargas libres en el interior del dieléctrico, calcular: a) el campo eléctrico, el desplazamiento eléctrico y el vector de polarización, cuando aplicamos una diferencia de potencial Vo entre las placas. b) las densidades de carga de polarización. c) la capacidad del condensador.



ε= εo 1+ y   d 21) Demostrar que en un dieléctrico lineal no homogéneo, puede existir una densidad volumétrica de carga ligada en ausencia de densidad de carga libre. Calcular su valor. Sol: -εo(E ·∇ εr)/ εr

ELECTRICIDAD Y MAGNETISMO. Electrostática-Medios materiales.

22) Si el espacio entre dos cilindros conductores coaxiales alargados está ocupado por un dieléctrico, ¿cómo debe variar la permitividad relativa con la distancia r al eje para que la intensidad del campo eléctrico sea independiente de r?. ¿Cuál sería la densidad volumétrica de carga ligada?. Sol: εr=K/r, ρb=λ/2πKr, siendo λ la densidad lineal de carga en el cilindro interior. 23) Un electrete tiene la forma de una lámina delgada circular de radio R y espesor t, polarizada permanentemente en la dirección paralela a su eje. La polarización P es uniforme en todo el volumen del disco. Calcular E y D sobre el eje, tanto dentro como fuera del disco. 24) Una esfera de radio a está formada por un dieléctrico homogéneo, con constante dieléctrica relativa εr. La esfera está centrada en el origen del espacio libre. El potencial eléctrico viene dado en el interior y exterior de la esfera, respectivamente, por: 3E R⋅cosθ Vin Comprobar que se cumplen las condiciones de contorno para el campo eléctrico y el desplazamiento eléctrico en la superficie de la esfera. E aRO 3 ⋅ εr −1 ⋅cosθ

Vout = −E RO

⋅cosθ+ 2

2 εr + 25) Desplazamos la carga 3 una distancia d/2 hacia la izquierda, manteniendo fijas las restantes cargas. ¿Es más estable la disposición anterior que ésta?. 1 2 3 4 •← d →• • •q -q q -q 26) Calcular la energía electrostática almacenada en el sistema del problema 4. 27) Calcular la energía electrostática almacenada en el sistema del problema 7. 28) Partiendo de una esfera de radio R a, que tiene una carga Q en la superficie, se inicia la acumulación de carga sobre una superficie esférica de radio R b (Ra>Rb), concéntrica con la anterior. Calcular el trabajo realizado para acumular sobre la superficie esférica de radio Rb una carga igual a Q/2. 29) Tenemos un sistema de cargas constituido por una distribución uniforme de carga Q en una esfera de radio R1 y otra de carga -Q distribuida uniformemente sobre una capa esférica, concéntrica con la esfera, de radio R2=5R1. a) Calcular el campo en función de la distancia al centro. b) Calcular la energía electrostática del sistema. c) Si quitamos la mitad de la carga -Q de la capa esférica, ¿cuál será la variación de energía electrostática del sistema?.

ELECTRICIDAD Y MAGNETISMO. Electrostática-Medios materiales.

30) Un condensador plano de superficie S y espesor d se carga mediante una batería con una diferencia de potencial Vo. Después de cargado desconectamos la batería. Sin tocar las placas introducimos una lámina metálica de espesor d/2. a) Calcular la densidad de energía electrostática antes y después de introducir la lámina metálica. b) Calcular la energía total en ambos casos. ¿En qué se ha invertido la diferencia entre las dos energías?. 31) Un condensador de armaduras planas, de superficie A=200 cm 2, separadas la distancia d=1 mm, tiene en su zona central una lámina de material dieléctrico, de la misma forma y tamaño de las armaduras, espesor de 0.6 mm y permitividad relativa εr=4, El condensador se ha cargado hasta adquirir entre sus armaduras el potencial V= 1000 V. Calcula: a) La capacidad del condensador. b) La carga del mismo. c) La energía almacenada. d) Los vectores desplazamiento eléctrico, campo eléctrico y polarización, representándolos gráficamente. 32) Una carga eléctrica Q se distribuye en una esfera dieléctrica de radio a y permitividad ε, de forma que las densidades de carga libre sean:

ρo(a / R) para 0 ≤ R ≤ a ρv =  0 para R ≥ a 

a) Expresar ρo en función de Q y a. b) Hallar la energía electrostática del sistema. 33) Dos planos conductores aislados infinitos, que se mantienen a potenciales 0 y Vo, constituyen una configuración en forma de cuña, como se ilustra en la figura. Determine las distribuciones de potencial en las regiones: a) 0 < φ < α , b) α < φ < 2π. Vo

α

θ

34) Calcular, mediante el método de las imágenes, la carga total inducida en una esfera conductora conectada a tierra, inducida por una carga puntual, Q, situada fuera de la esfera, a una distancia D de su centro.

ElectrostÆtica

PROBLEMAS DE ELECTROST`TICA MEDIOS MATERIALES  Ejercicio

1

Existe simetr a esfØrica, por lo cual podemos hacer el problema mediante el teorema de Gauss. Calcularemos primero el campo electrostÆtico y luego el potencial. Sesfera = 4πR2

: R < Ri

a) Regin 1 Q

∫E ⋅ ds =ε

0

∫ E ⋅ ds = ∫ E ⋅ ds ⇒ E paralelo a ds = ∫ E ⋅ds = E∫ds = E ⋅ S ES

E

R ⇒

 E1 =

1 Q  V a 2 ⋅ m 4πεR 0 R

( )

Regi n 2: Ri Ro.

 E1 =

1 Q  V a m 4πεR 2 ⋅ 0 R b) Para calcular el potencial en todos los puntos del espacio se integra el campo elØctrico. Empezaremos desde la regi n 3 hacia la 1. Usaremos la ecuacion:

( )



V = − ∫E dR⋅ + K

(con K = cte de integracion)

( )

V = −∫ 4π1ε0 ⋅ RQ2 dR = 4πQε0 R + K( )V⇒

V3 = 4πQε0 R + K V

Para hallar la constante, partimos de la suposici n de que el campo en el infinito es cero; igualando, tenemos K⇒K=0

0 Regi n 2:

El campo en esta regi n es cero, luego el potencial es constante. Por la continuidad del potencial, Øste toma el valor de la regi n 3 haciendo R=Ro:

V2 = 4πεQ0 Ro

( ) V

Regi n 1:

dR ⇒V

V

K

V

Para hallar K, por la continuidad del potencial, basta con igualar el valor del potencial de la regi n 1 de valor Ri al de la regi n 2:

Q

+K=

Q

⇒K=

Q



Q

⇒K=

Q1

1

ElectrostÆtica 

4πε0 Ri

4πε0 Ro

4πε0 Ro

4πε0 R0 − Ri  ⇒

4πε0 Ri

V1 = 4π ε πε1R ⋅ Q0 + 4 Q 0 R10 − R1i  ⇒ V1 = 4πεQ 0 R1 + R10 ( V − R1i 

)

Ejercicio 2 a)

Suponemos la longitud del tubo lo suficientemente grande como para que E sea perpendicular al eje. Entonces, las tapas no contribuyen al calcular con el teorema de Gauss. Zona 1: ; Q = ρl ⋅L

∫ E ⋅ds = ∫ E ⋅ds + ∫ E ⋅ds = ∫ E ⋅ds ⇒ E paralelo a ds = ∫ E ⋅ds = E∫ds = E⋅S

ElectrostÆtica tapas

lateral

lateral

S = 2πrL ⇒ E ⋅2πrL = ρl ⋅ L

⇒E1 = 2περrl 0 ar  

ε0

Zona 2: Esta regi n es el interior del conductor, por tanto no existe campo electrostÆtico: E2 = 0 ar Zona 3: Como el metal es neutro, la carga total es nula (se induce una carga igual y opuesta a la del hilo en la superficie interna, e igual a la del hilo en la superficie externa). La carga encerrada por la superficie gaussiana queda igual a la del hilo. Por tanto, en esta regi n el campo es el mismo que en la zona 1:



ρl ar 

E1 = 2πrε0



Una vez que hemos calculado el campo de las 3 zonas, simplemente damos valores y hallamos el valor numØrico del campo en cada zona: r = 1m (Zona 3)  E = 60×10−12 2πε0 r = 0.25 m (Zona 2) r = 0.15 m (Zona 1)

30×10 12 =

0



(V m)

πε

 E2 =0 V m

( )

 60 ×10 −12 0.2 ×10 −9 V E1 = m 2π ε 0 ⋅ 0.15 = π ε 0

( )

b) Nos piden la diferencia de potencial entre ambas superficies del conductor. Sabemos que el potencial en el interior de un conductor es constante, asi que la diferencia de potencial entre ambas caras es nula: Vba = 0

ElectrostÆtica 

Siendo Vba la diferencia de potencial entre cada cara.

ElectrostÆtica Ejercicio 3 Tenemos dos esferas :

2

b2 b1 1

Como la carga se distribuye uniformemente en la superficie, el campo fuera de las esferas es el mismo que el produce una carga puntual colocada en el centro de valor la carga de la esfera respectiva. Aplicando el teorema de Gauss, tomamos una superficie gausiana de radio φ:

∫E ⋅ ds =ε Q , que operando nos queda: E= 4πεφQ 0

o

2

 r a

Para la esfera 1: E1 = 4πεq1ob12 ar  

q

Para la esfera 2: E2 = 4πε 2ob22 a

r

Donde q1 y q2 son las cargas en la superficie de cada esfera. AnÆlogamente, integrando el campo elØctrico, el potencial fuera de las esferas es anÆlogo al de una carga puntual. Por la continuidad del potencial, su valor en la superficie de las esferas (igual que el del interior, por ser conductores) serÆ respectivamente:

Para la esfera 1: V 1 =

q1

4πε0b1 Para la esfera 2: V 2 =

q2

4πε0b2 El problema nos dice que entre las dos esferas hay una carga encerrada q, es decir: q = q1+ q2. Al estar unidas por un cable, el potencial en ambas esferas es el mismo: V 1 = V2. Lo que nos lleva, despejando, a que q1b1 = q2b2Teniendo en cuenta que: q = q1+ q2 obtenemos dos expresiones de la carga en la superficie de cada una de las esferas, en funci n de datos conocidos como son el radio y la carga total:

ElectrostÆtica 

q1 =

b q1

b

q

2 Y q2 = b1 +b2 b1 +b2 La carga estÆ repartida uniformemente, es decir, las esferas poseen densidades constantes de valor:

ρσ1 = q1 / s1 , ρσ2 = q2 / s2, b12 , s2 = 4πε0 b22

donde s1 y s2 son las superficies de las esferas: s 1 = 4π ε0

Sustituyendo datos, obtenemos dos expresiones en funci n tambiØn de datos conocidos. q

Densidad de la esfera 1: ρ01 =

4πb b1( 1 +b2) q

Densidad de la esfera 2: ρ02 =

4πb b2( 1 +b2) Y si queremos dejar el campo elØctrico en funci n de la carga hallada: q

Para la esfera 1: E1 = 4πε0b b1 ( 1 +b2) ar q



Para la esfera 2: E2 = 4πε0b b2 ( 1 +b2) a

r

ElectrostÆtica Ejercicio 4

R

R3 R2

R4 R1

a) Para hallar la distribuci n de cargas hay que recordar que las cargas en los metales se distribuyen en la superficie. AdemÆs, si se aplica el teorema de Gauss a una superficie en el interior de un metal, al no existir campo elØctrico en Øste, la carga total en el interior de la superficie gaussiana ha de anularse. Sea r la distancia al centro de las esferas: r=R tenemos una distribuci n Q y su densidad es ρ = −

Q +Q

2πR3L r = R1 tenemos una distribuci n Q y su densidad es ρ =

−Q

2πRL r = R2 tenemos una distribuci n Q + Q y su densidad es ρ =

Q +Q

2πR2L r =R3 tenemos una distribuci n (Q+Q ) y su densidad es ρ = − r =R4 tanto ρ = 0

Q +Q

2πR3L debido a la toma de tierra, no existe carga y su densidad es por

ElectrostÆtica 

b) Para hallar el campo elØctrico aplicamos la ley de Gauss a superficies cil ndricas coaxiales en las distintas regiones, considerando la simetr a cil ndrica del problema. As ,

∫ E•ds= (E||ds) = (E cte en s)

r

SE

S

ds E → E2

(Teorema de Gauss)

Por lo tanto: r 2 π Regi n 2 => E2 = 2π εbLQ 2 ar

εQaL ar= 4 Q ar

2π εbL 2

1



Y como se ha de cumplir que E



= 4E 2 :

1

Q

a

2π εaL 1R = 4 2π εbLQ2 aR

a

Y despejando ε2 nos queda que: ε2 = 4ε1 b Sabiendo que b = 2a y que ε1=4ε0 :

ε2 =16ε0 2aa = 8ε0

ElectrostÆtica b)Para hallar la capacidad de la estructura utilizaremos la siguiente ecuaci n: C =

Q

,

∆V siendo Q la carga total almacenada en el condensador y ∆V la diferencia de potencial entre las placas, que para nuestro caso serÆ Vo. Como consecuencia, debemos relacionar la diferencia de potencial (Vo) con la carga en las placas. Para ello nos ayudaremos de la siguiente ecuaci n:



vv12

dV = − ∫ E dl⋅  C

Tomando como curva C para nuestro ejemplo, la direcci n radial entre los conductores. Debido a que no estamos en el vac o, sino que la zona que hay entre las placas esta constituida por dos dielØctricos, con campos distintos, debemos separar la integral en dos: una integral para la Regi n 1, que va desde R = a hasta un punto R = c (donde termina la regi n 1 y empieza la regi n 2) y otra integral que va desde R = c hasta R = b para la Regi n 2:

Vo = −C∫E dl⋅ = ∫ac E dR1 ⋅ + ∫bc E2 ⋅dR



Como dl y el campo son vectores paralelos y con el mismo sentido, nos queda que :

Vo = ∫ac E1 ⋅dR +



bc

E2 ⋅dR = ∫ac 2π εQRL 1 ⋅dR +



bc

dR Resolviendo nos queda : Q

Vo = 8επ

o



c

1

2 a

L ⋅ ln a + 2 ln c



Sustituyendo esta ecuaci n en la expresi n de la capacidad, se tiene que: C = ∆QV = VoQ = 8επo L⋅ln ac + 12 ln 2ca −1

⋅2π εRLQ 2

ElectrostÆtica Por lo que la capacidad por unidad de longitud serÆ:

C 8επo⋅ln ac + 12 ln 2ca −1 L  Ejercicio

10 La capacidad de un condensador viene dada por la siguiente expresi n: C = Q ∆V donde Q es la carga total del condensador y ∆V la diferencia de potencial entre las placas. Para calcular la diferencia de potencial utilizaremos la siguiente expresi n:



dV =−∫E dl

vv12

 ⋅

C

Debemos pues de calcular el campo elØctrico en el condensador. Para ello utilizaremos la expresi n D=εE, de donde despejando Enos queda que E= D . ε Para hallar el vector desplazamiento elØctrico entre las placas (fuera es nulo), utilizamos la ley de gauss para medios materiales:

∫ D ds⋅ = ρ

v

s

Tomaremos como sistema coordenado, las coordenadas esfØricas, por la simetr a esfØrica del problema y como superficie gaussiana una esfera concØntrica de radio R. 



Para calcular el flujo, ds y D tienen la misma direcci n y sentido (radial), por lo que el producto escalar de estos dos vectores serÆ igual al producto de m dulos. Como D en esa zona es constante en la superficie gaussiana :

ElectrostÆtica

∫



D

⋅ds

= D ds∫



= D S⋅ =

D4πR2 s

s

Siendo 4πR2 la superficie de la esfera de radio R. Como consecuencia, la carga total encerrada en esa superficie gaussiana es Q.

As , la ley de gauss nos queda de la siguiente forma: D4πR2 = Q

Q

Y despejando D, obtenemos el m dulo del vector desplazamiento: D = 4π R2 

DÆndole carÆcter vectorial:

D=

Q

2

a R

4πR =D

Ahora hallaremos el campo elØctrico en la regi n: E



=Q a 2

R

ε επ4 R

Sustituiremos ahora la permitividad absoluta por su verdadero valor: ε=εε0 r , siendo ε0 la a

permitividad del vac o y εr la permitividad relativa que en nuestro caso serÆ: R . El campo elØctrico en la regi n es entonces :

E= εεπQ4 R2 aR =aQ4 R2 aR = ε π0aQ4 R aR 0 r

ε π0 R

Calcularemos ahora, la diferencia de tensi n entre las placas. Para este ejercicio nuestra curva serÆ la recta que va en direcci n radial desde R1 a R2, por lo que la ecuaci n nos queda:

ElectrostÆtica Vo = C∫E dl⋅ = C∫ε π0aQ4 R aR ⋅dRaR = C∫ε π0aQ4 R ⋅dR = ε π0aQ4 ln RR12

Donde hemos integrado a lo largo de la direcci n radial, siendo E y dl= dRaR vectores paralelos.

Sustituyendo el potencial, en la f rmula de la capacidad C =

Q

nos queda:

∆V

C = ∆QV

= VoQ

= ε0aR4π2 ln

R1  Ejercicio 11

Carga puntual

Q

Permitividad del dielØctrico (εrε0) - Nos pide hallar el vector de polarizaci n para ello partiremos de la siguiente f rmula: D= ε0E+ P Y de esta f rmula despejamos el vector de polarizaci n, nos queda: P= D−ε0E Luego nuestro problema queda reducido a hallar el vector de desplazamiento elØctrico y el campo elØctrico, y sustituir dichos valores en la ecuaci n anterior. a) Hallar el vector de desplazamiento elØctrico Para hallarlo aplicaremos el teorema de Gauss para dielØctricos:

∫D• dS= Q S

v

ElectrostÆtica

(Cargas libres encerradas, en nuestro problema es la carga puntual Q) Para resolver dicha ecuaci n tenemos que elegir una superficie gaussiana, vamos a escoger una esfera de cualquier radio r. Ya sea dentro o fuera de la esfera la carga libre encerrada siempre es Q: Vemos que por simetr a el campo elØctrico solo depende de la coordenada r, y Østa a su vez es constante, por lo tanto puede salir de la integral, y al ser los vectores paralelos, podemos eliminar el carÆcter vectorial y dejar la f rmula de la siguiente manera: D(r)∫dS = Q S

Por lo tanto el vector desplazamiento nos queda: D DÆndole carÆcter vectorial: D= Q 2 ar 4πr

∀r

Representaci n grÆfica del vector desplazamiento  D

Q 4πR 2 R

r

El problema nos pide que lo representemos con dielØctrico y sin Øl, pero en este caso la grÆfica ser a la misma. b) Hallar el campo elØctrico Si suponemos el medio lineal podemos escribir que: D=

εELuego el campo elØctrico serÆ:

ElectrostÆtica Epara r < R D= εE

E= εD= 4πQεr 2

ar E para r > R D= ε0EE= εD0 = 4πεQ0r 2 ar  Representaci n grÆfica del campo elØctrico GrÆfica con esfera dielØctrica E Q 4πε0R2 Q 4πεR2 r R El campo elØctrico es discontinuo en r = R, debido al cambio de permitividades. Sin embargo sin esfera dielØctrica el campo si es continuo como se puede apreciar en la siguiente grÆfica. (campo creado por la carga puntual Q).  E

Q 4πε0 R 2 R

r

DespuØs de obtener los datos que nos eran necesarios para hallar el vector de polarizaci n, lo œnico que nos queda es sustituirlos en la ecuaci n:

ElectrostÆtica P= D−ε0E

Ppara r > R

P

 =0

Esto se debe a que en r > R nos encontramos en el vac o y ah el vector de polarizaci n es 0. Ppara r ≤ R

 4πε 4πr

4πr  ε0εr

ε



r

r

- Lo otro que nos ped a el problema eran las densidades de cargas ligadas: Densidad de carga de polarizaci n en volumen

R La carga volumØtrica de polarizaci n es cero. Densidad de carga de polarizaci n superficial

ρPS = PS • ar = πQR2   εrε−r 1 4 Como la carga de polarizaci n total en el dielØctrico ha de ser nula, existe una carga puntual de polarizaci n, QP, en el centro del dielØctrico de valor:

Qp = −ρPS ⋅ 4πR2 = Qεrε−r 1 - Lo œltimo que nos queda es hallar el potencial para poder dibujarlo, que es otra de las cosas que nos piden.

V = −∫E



• dr



+ Cte

ElectrostÆtica

V para r > R

V = −∫ 4πεQ0r 2 dr = 4πεQ0r + Cte Como sabemos que: V(∞) = 0 → cte. = 0 Y nos queda que el potencial para r > R es: V=

Q 4πε0r

V para r < R

V

Cte

Por la continuidad del potencial en la superficie V(rR) en R Pues si igualamos las dos f rmulas y despejamos la Cte, podemos hallar su valor, que es el siguiente:

Cte = 4πQRε10 − ε1

Y si ese valor lo sustituimos en el potencial para r a

a

D = εεr −r 14πK Ra2 aR 

ElectrostÆtica V = −∫εK(ε⋅rε−r1) Ra2 dR = ε0K(ε⋅rε−r1) Ra + C 0

Obtenemos C haciendo tender R → ∞: V(

)

C⇒C=

0 0

r

0

r

Por tanto, el potencial en el exterior de la esfera dielØctrica queda: ⋅ε r − 1

Vexterior = ε0K(ε r

) Ra

R< a V

C

Hallamos C haciendo R C⇒

V

K ⋅ε K ⋅

ε

C⇒

⇒ C = εK(ε⋅rε−r 1) +ε(Kεr⋅ε−r1)lna ⇒ C = (εKr ⋅−ε1r )ε10 + lnεa 

0

Sustituyendo C y simplificando: V = −ε(Kεr⋅ε−r1)ln R + (εKr ⋅−ε1r )ε10 + lnεa  = (εKr ⋅−ε1r ) ε10 + lnεa − lnεR  = ×εε 

(

K ⋅ ε r + ln a R R K ⋅ε εr +1lna = {ε =ε 0ε r } = ε (ε − ) ε (ε −1) r

(

)

r

0

)

r

Por tanto, el potencial en el interior de la esfera dielØctrica es:

(

K⋅ ε V interior

=

r

)

+ lnaR

ε0 (εr −1)

ElectrostÆtica

d) Representar grÆficamente la variaci n de potencial con la distancia: V(R) 4 3,5 3 2,5 2 1,5 1 0,5

R

0

Variación del potencial con la distancia

a

 Ejercicio

13

V(Ra)

ElectrostÆtica

a) Densidades de carga de polarizaci n:

ρps : densidad superficial de carga de polarizaci n equivalente densidad volumØtrica de carga de polarizaci n equivalente 





ρps = P⋅ a R = 2 ⋅10−6 a z ⋅ a R = 2 ⋅10−6 cosθ C/m2

ρps

ρ



ρpv

=

pv

= 2



10−6 cos

cte



θ

C/m2



P



=0

= −∇

P

b) Potencial elØctrico en el centro de la esfera

Vint = 4πε1 o ∫s’

ρrps ds’ + 4πε1 ∫v’ o

ρrpv dv’

ρpv :

ElectrostÆtica En nuestro caso, como ρpv= 0:

Vint =

ds’

En coordenadas esfØricas tenemos que: S’ ; r = R ds’= r 2 senθdθdφ Por tanto,

Vint = 4πε1 o ∫0πρrps r 2senθdθ∫02πdφ= 4πεR o ∫0π2⋅10−6 cosθ⋅ senθdθ∫02πdφ=

R ⋅10−6 =

10−6 (−cosθ)]π = 0

π

c) Demostraci n de que la carga libre en el dielØctrico es nula: El dielØctrico es simple, o sea, lineal, homogØneo e is tropo, por lo que la permitividad relativa es constante ( εr = cte ). Sea ρv la densidad de volumen de las cargas libres. Sabemos que se cumple que:  

∇ D 

=ρv



D =ε0 E + P

 

ε0∇ E

 

+∇ P

= ρv



D =ε⋅E

ρρv = −∇P

ε0∇ D −ρpv = ρ⇒εε0 ∇D= εε0 ρv = ρv +ρpv ε

ε

ElectrostÆtica = ε0 −ερpv = 0

ρv

Vemos que ρv = 0 debido a que ρpv = 0.  Ejercicio 14 El problema nos dice que tenemos un dielØctrico al cual se le a hecho una cavidad esfØrica, como el de la figura, sin embargo, no especifica si el dielØctrico es finito o infinito de manera que vamos a considerarlo finito por hacer el problema mÆs ceæido a la realidad. De esta forma el campo elØctrico en el interior del dielØctrico es generado por un

campo

exterior

al



dielØctrico ( E ext ) que, por las condiciones de contorno y suponiendo que solo tiene componente normal (a Z ), obtenemos de la siguiente forma:

Segœn las condiciones de contorno la componente normal del vector desplazamiento se conserva: D

 n

=D



dn

ext

Segœn la relaci n entre el vector Desplazamiento y el campo elØctrico, la componente normal del campo elØctrico externo vale:

ε0 ⋅ Eext =ε⋅ Ed n

n

Eextn = εε0 Edn  Como solo tiene componente normal el campo del dielØctrico, el campo exterior solo tendrÆ componente normal:  Eextn = Eextn ⋅ aZ = Edn ⋅ aZ

εε0

ElectrostÆtica Para hallar el campo en el centro de la cavidad podemos hacerlo de varias formas. Yo he elegido utilizar el teorema de superposici n, de esta forma, el campo en el centro de la cavidad es la suma del campo exterior con el campo producido por cargas de polarizaci n. Debido a la interacci n del campo elØctrico con las molØculas del dielØctrico, se crean cargas de polarizaci n en la superficie interior del dielØctrico: Estas cargas se representan

-

-

-

+

+ +

vectorialmente con el vector de P) que, viene relacionado polarizaci n ( con el campo elØctrico segœn la

+

+

siguientes expresiones:

D=ε0 Ed + Pε0 Ed + P=εEd P = ( ε−ε0 )E d

D=εEd  P= (ε−ε0 )Ed

Lo cual significa que el vector polarizaci n tiene la misma direcci n y sentido que el campo elØctrico. El campo elØctrico Ed es uniforme (constante segœn la posici n), lo cual significa que su divergencia es nula. Esto implica que la divergencia de D tambiØn es nula y por consiguiente que no existe densidad de carga libre volumØtrica dentro del dielØctrico (no existen cargas libres en el interior del dielØctrico): E= cte⇒ D= cte⇒ ∇⋅ D= 0 ⇒ρv = 0 De la misma forma la densidad volumØtrica de carga de polarizaci n la hallamos como: ρ = −∇⋅ P πv

ElectrostÆtica Vemos que se anula igualmente debido a la uniformidad del campo. Esto va a simplificar un poco la expresi n que nos da el campo elØctrico creado por las cargas de polarizaci n: 0  E p (0) = 4πε1 0 ∫v’ ρRpv2 aRdv’+∫s’ ρRps2 aRds’ Tenemos que hallar la densidad superficial de cargas de polarizaci nρps a lo largo de la superficie interna del dielØctrico. Para ello utilizamos la siguiente expresi n: ρps = P⋅ an Tenemos que por lo tanto hallar el producto escalar de ambos vectores para lo cual antes los definiremos:

Se deducen de las f rmulas que hallamos antes, que el vector polarizaci n es paralelo al campo elØctrico de manera, que tendrÆn la misma componente vectorial. Con respecto al vector radial, decir que es un vector que va perpendicular a la superficie del dielØctrico y hacia fuera de Øste. Ahora que tenemos el vector radial lo que hacemos es pasarlo de coordenadas esfØricas a cartesianas para poder integrar si problemas. Para ello hacemos uso de proyecciones y de trigonometr a elemental. a

R == P − ⋅

sen

aZθcosφ⋅ aX − senθ⋅ senφ⋅ aY − cosθ⋅ aZ

P Ya tenemos definidos ambos vectores, ahora hacemos el producto escalar:

ρps = P ⋅ an = P ⋅ aR = P cosα = P cos(180”−θ) = −P cosθ Con los datos que tenemos, ya podemos hacer la integral que nos darÆ el campo producido por las cargas de polarizaci n:

ElectrostÆtica aRds’ =

ds’

Necesitamos definir nuestra superficie de integraci n as como el diferencial de superficie a utilizar que, al estar trabajando con simetr a esfØrica, serÆ el mismo expresado en coordenadas esfØricas: ds’ = R 2 senθ⋅ dφ⋅ dθ La superficie es una esfera completa, de manera que debemos integrar segœn los l mites siguientes: s ’  eje de

al

θ π20π

Hay que fijarse que θ estÆ definida de 0 a π. Esto se debe

 



φcoordenadas definido para hacer el problema.

0

Continuamos pues con nuestra integral:

E sen

=−

p

⋅aX −

−

⋅ sen sen a⋅ ⋅ Y −⋅aZ  ⋅

⋅R sen d2 θ φ θ⋅ senφ]

02π=0

 − 0

sen1θ =0 0

cos3θ

(ε−ε )E =

0



4πε00d 2π

13aZ = (ε−3εε00)Ed aZ

3

 a

π Z

= (ε−2εε00)Ed 13 +

1







⋅ =



π



ElectrostÆtica

Por lo tanto el campo creado por las cargas de polarizaci n en el centro de la cavidad es:  EP (0) = (ε−εε00 )Ed aZ 3

Ahora, por superposici n, sumamos los campos exterior y el creado por las cargas de polarizaci n y as obtenemos el campo total en el centro de la cavidad:

Eint (0) = EP (0) + Eext (0) = (ε3−εε00 ) Ed + ε

ε0 Ed aZ = 4εr3− 1 Ed ⋅ aZ

ElectrostÆtica  Ejercicio

15

Utilizando las condiciones de contorno para los campos electrostÆticos tenemos: LA COMPONENTE NORMAL DEL CAMPO: D1N -D ε

2N

=ρS ; como en la interface no hay carga libre: ρS = 0 ⇒ D1N = D ε

= 2 ⋅ Ε2N ⇒ 1 ⋅ Ε1 ⋅ cos

α 1

=

ε

⋅ Ε2 ⋅ cos

2

α 2

ε 2N

1

⋅ Ε1N

(ecuaci n 1)

LA COMPONENTE TANGENCIAL DEL CAMPO: Sabemos que Østa se conserva Ε1T = Ε2T ⇒ Ε1 ⋅sen

α 1

= Ε2 ⋅sen

α 2

(ecuaci n 2)

Para hallar la direcci n del campo en el medio 2, tendremos que averiguar α2 y para ello lo mÆs sencillo es dividir la ecuaci n nœmero (2) entre la ecuaci n (1):

1

1

ε tgα1 = ε 2 tgα2 ⇒ tgα2



2 ε1 tgα1 . Despejando:

1

y esta serÆ la direcci n que estÆbamos buscando

α2 = arctg εε12 Existen dos formas posibles de calcularla magnitud del campo 2: tgα1  a) Operando con las ecuaciones (1) y (2): elevando las dos  ecuaciones al cuadrado y multiplicando la (2) por ε22 , y nos quedan las siguientes expresiones:

ε12 ⋅ Ε12 ⋅cos2α1 =ε22 ⋅Ε22 ⋅cos2α2

ε22 ⋅Ε12 ⋅sen2α1 =ε22 ⋅Ε22 ⋅sen2α2

ElectrostÆtica

Si ahora sumamos las dos expresiones tenemos: Ε12 (ε12 cos2 α1 +ε22sen2α1 ) = ε22Ε22 , donde en el segundo miembro de la ecuaci n se tuvo en cuenta que: sen 2 α2+ cos 2 α2= 1. Por œltimo, despejando Ε2 : Utilizando el teorema de pitÆgoras

2 2 2 2 2 1 Ε2 = Ε 2 N + Ε2T = (Ε 2 senα 2 ) + (Ε2 cosα 2 ) = (Ε1senα 1 ) + (ε

ε

2

Ε1 cosα 1 )

2

2

 ε1 cos  Ε 2 = Ε1 ⋅ sen α 1 +   2 ⋅ α1   ε   2

y comprobamos que nos da el mismo resultado. 16

 Ejercicio

El problema especifica que se desprecien los efectos de borde. Esto quiere decir que las lÆminas estÆn muy juntas y se tratan como planos infinitos, con lo que podemos suponer que el campo en la regi n interna del condensador es uniforme. Si hubiØsemos considerado el efecto de borde el campo no hubiera sido uniforme en los extremos de las placas. Al tener una diferencia de potencial, entre las placas ambas quedan cargadas con una carga +Q y Q respectivamente.

ElectrostÆtica

Campo Aplicando Gauss a la primera placa (+Q) para conocer el aporte que realiza al campo de la regi n interna:

∫E ds = ∫E ds + ∫ E ds E ds = 2∫ E ds = 2E ∆S 1

11

12

S

2E1∆S = ε E1 2ρεso ax

12

S1

Qvo

S2

12

+ ∫E ds1

1

S3

S2

S2

,2E1∆S = ρεs∆o S ,E1 = 2ρεso

=

El razonamiento a seguir con la segunda placa (-Q) es anÆlogo, de manera que:

3

= 2∫

ElectrostÆtica E2 = 2ρεso ax = E1

y sumando los aportes de la placa (+Q) y (-Q), obtenemos que el campo en la regi n interna del condensador es:

ρs a x = E εo E1 + E2 =

Densidad de carga Para calcular la densidad de carga superficial en las placas nos ayudamos de que ya conocemos el potencial (es un dato del problema), conocemos el campo y conocemos la expresi n que relaciona campo y potencial:

Vo = − Q∫



Edl = − Q∫ −Edl = E

∫ dl = El

Q

−Q

ρ

E = Vo , s = Vo , l ρs =

εo l

−Q

−Q

V

lεo

o

Energ a La energ a se puede calcular de dos maneras. Una de ellas es utilizando la f rmula de la energ a en un condensador: W = QV

ElectrostÆtica que utilizaremos mÆs adelante. El segundo camino que podemos seguir para su cÆlculo es la utilizaci n de la f rmula general:

W DEdV teniendo en cuenta que el medio es lineal, es decir: D =εE Resolviendo por este mØtodo:

12 ∞∫εoE2dv = 12 dentro∫ εoE dv2= 12εoE2 dentro∫ dv = 12εo ρεos22 Sl = SV2 o2 l εo =W

Capacidad

C = VQ ,C = VQo ,C = ρ ερssSl =

o

lS = C

εo b) Manteniendo la diferencia de potencial que exist a entre las placas del condensador se introduce ahora un dielØctrico de la siguiente forma:

ElectrostÆtica Al polarizarse el dielØctrico se anulan las cargas de los dipolos interiores y s lo quedan las cargas externas en la superficie del mismo. Estas cargas no son libres, son cargas ligadas. Al haber introducido un dielØctrico en el interior del campo manteniendo el potencial constante, obtendremos una variaci n en la densidad de carga que contrarreste el efecto del dielØctrico.

Campo Sabemos que una de las condiciones de contorno en la superficie que separa dos medios es: D1n − D2n = ρs Aplicando esto a las caras externas del dielØctrico, y teniendo en cuenta que la densidad de carga libre en su superficie es 0, obtenemos la siguiente condici n de contorno para las dos superficies que separan el dielØctrico de los huecos: D1n = D2n ,D2n = D3n y por tanto: D1n = D2n = D3n

AdemÆs en este caso D1 = D1n,D2 = D2n ,D3 = D3n , al ser los vectores D perpendiculares a las superficies entre medios. De esta forma s lo tendremos que calcular uno de los D para conocerlos todos. Calculamos D1 :

ElectrostÆtica

Suponiendo que el flujo de campo elØctrico en S3 es 0 al anularse el flujo producido por +Q con el de Q:

∫Dds = ∫Dds S

S

D S1∆= Q D Sv , 1

1

2

= ∫D ds1

2

= D S1∆

S



= ρs∆S D,

1

= ρ s ⇒ D1 = D2 = D3 = ρ s

de aqu obtenemos: D1 =ε ρε ρE1 =s , 0E1 = s , D2 =ε ρεε ρE2 =

s

E1 = ερ0s

, 0 r E2

D3 =ε ρε ρE3 =s , 0E3 = s ,

E2 = = ερ0εsr

s

,

E3 = ερ0s

Densidad de carga La hallamos a travØs del potencial, que sigue siendo el dato del problema, y la expresi n que ya conocemos del apartado a) que nos relaciona campo y potencial V = El :

ElectrostÆtica Vo = E1 4l + E2 2l + E3 4l = ε εε ερ ρ ρ ρos 4l +

o sr

2l +

os

4l = 2ε ε ε εslo

1+ 1r  = ρsol ε2r +r1⇒ 

⇒ρs = Volεo  ρ ρs = ε2rε+r1, +r1

s0

2ε εr

 siendo ρs la densidad de carga en las placas antes de introducir el dielØctrico (la del apartado a) ) o

Energ a Utilizaremos ahora la f rmula de la energ a de un condensador:



W = 12QV = 12ρsSVo = V S2o Volεo

W = Wo ε2rε+r1

siendo Wo la energ a del condensador dielØctrico (la del apartado a) )



ε2rε+r1,

antes

de

introducir

el

Capacidad

C = Q ,C = Q ,C = ρ ε εsS =oS ε2r V

Vo

Vo

l

Co ε2rε+r 1 = C 

+r1 =

siendo Co la capacidad del condensador antes de introducir el dielØctrico (la del apartado a) )

c) En este apartado consideramos que, antes de insertar el dielØctrico en la regi n interna del condensador, la fuente de tensi n ha sido desconectada. De esta manera la carga que ya exist a en las placas va a permanecer constante (igual que en el apartado a) ) y lo que cambia ahora para contrarrestar el efecto del dielØctrico es el potencial.

ElectrostÆtica

Campo El campo en los huecos y en el dielØctrico va a seguir teniendo las mismas expresiones que calculamos en el apartado b): Ehue = ερos ,Edie = εερo sr Densidad de carga Se mantiene igual que antes de insertar el dielØctrico (apartado a) ):

ρs =Vol

ε

o

Energ a Para calcular la energ a por la f rmula de la energ a en un condensador necesitamos conocer el potencial. Lo calculamos a partir de la relacion V = El y las expresiones conocidas de E : V = E1 4l + E2 2l + E3 4l = ε εε ερ ρ ρ ρos 4l +

o sr

ε ε εslo 1+ 1r  = ρsol ε2r +r1 =

2l +

os

4l = 2ε

Vo ε2rε+r 1 = V 

ElectrostÆtica as tenemos:

W = 1 QV = 1ρsS ρε εsol 12+εr r  = V S022lεo 12+εεr r  = 2

2

Wo 12+εε r r  = W

siendo Wo la energ a del condensador antes de introducir el dielØctrico (la del apartado a) ) Capacidad

C = VQ ,C = Vo 1Q2+εεr  = ρVsS 12+εεr r  = VolVεooS 12+εεr r  = εolS 12+εεr r  = r

o



C0 12+εε r r  = C



siendo Co la capacidad del condensador antes de introducir el dielØctrico (la del apartado a) )

ElectrostÆtica

 Ejercicio 17

y

(1)

(2) 0.01d

a)

d

Antes de la fractura :

Los dos condensadores son iguales y estÆn sometidos la misma tensi n, entonces: E1 = E2 = Vo / d ( - ay) D1 = D2 = ε E = ε ( Vo/d ) = 100 εo ( Vo/d) ( - ay) b)

DespuØs de la fractura: Como la fractura se produce despuØs de haber desconectado la bater a, la carga total en las placas se conserva. Qt antes = Qt despuØs Hallando la carga antes de la fractura : Qa1 = Qa2 = ρ S = ε E S = 100 εo ( Vo/d ) Qtotal = Qa1 + Qa2 = 200 εo S ( Vo / d )

(1)

Mientras que para la carga despuØs de la fractura se ha de considerar que en el condensador 1 existen dos campos elØctricos: Ed1 (campo en el dielØctrico) y Ef1 (campo en la fractura), que vienen dados por: Qd1 = ε Ed1 S Qd2 = ε E2 S Con lo que la carga total serÆ: Qtotal = Qd1 + Qd2 = ε S ( Ed1 + E2 )

(2)

ElectrostÆtica Igualando las ecuaciones (1) y (2) : 200 S εo ( Vo / d ) = 100 εo S (Ed1 + E2 ) As , (Ed1 + E2 ) = 2 ( Vo/d ) (3) AdemÆs, si llamamos V a la tensi n de los condensadores tras la fractura, se ha de cumplir que: V = Ed1 d + Ef1 0.01d V = E2 d Para (1) y (2) respectivamente. Por tanto: Ed1 d + Ef1 0.01d = E2 d Ed1 + Ef1 0.01 = E2

(4)

Por otro lado, aplicando la continuidad del desplazamiento elØctrico entre el dielØctrico y la fractura del condensador (1), asumiendo que no existen cargas libres superficiales, y teniendo en cuenta que los vectores desplazamiento elØctrico son normales a las placas: Dd1 = Df1 ε Ed1 = Ef1 εo 100 εoEd1 = Ef1 εo 100 Ed1 = Ef1 (5) Las expresiones (3), (4) y (5) forman un sistema de tres ecuaciones con tres inc gnitas. Si las resolvemos nos da como resultado: Ed1 = 2Vo/ 3d ( - ay) E2 = 4Vo / 3d ( - ay) Ef1 = 200Vo / 3d ( - ay) Dd1 = 100 εoEd1 = 200 εo Vo / 3d ( - ay) Df1 = 100 εoEf1 = 400 εo Vo / 3d ( - ay)

b) El potencial entre las placas lo podemos calcular en el condensador (2) como: V = E2 d = 4Vo d/ 3d = 4Vo / 3  Ejercicio 18 Un esquema de la secci n transversal del cable coaxial es el mostrado en la figura. Las especificaciones de partida son:

ElectrostÆtica • En el conductor 2: o ri = 2•10-3 m o V = Vi • En el conductor 2: o V = Vo • En el dielØctrico: o εr = 2.6 o Rig. DielØctrica = 20•106 V/m

o Emax = 0.25 •

20 •106 = 5 • 106 V/m • Vi Vo = 10 kV = 104 V

Lo primero que debemos hacer es hallar como se distribuye el campo elØctrico en el dielØctrico. Aplicando el principio de superposici n, este campo es la suma del campo creado por el conductor interior y el creado por el conductor exterior: E = E + E1 2 elØctrico: Calculemos D= D + D1 primero 2 , que generan ambos conductores,

z

y el vector desplazamiento

utilicØmoslo para calcular el campo. Para ello eliminemos

D

r

dS

primero el conductor 2. El desplazamiento D1 podemos hallarlo aplicando la generalizaci n de la Ley de Gauss: D dS = Q

⋅  v

∫

s

D dS

ρs

h

, donde S es una superficie cerrada

cualquiera y Q es la carga libre

v

encerrada dentro de la

superficie. As , considerando que el conductor 1 tiene una densidad superficial de carga

ρs , elegiremos como superficie gaussiana un cilindro coaxial al conductor, de radio r y altura arbitraria h . Por simetr a, y como suponemos que el medio circundante (dielØctrico) es simple, el vector desplazamiento elØctrico serÆ radial y su m dulo s lo dependerÆ de la distancia r al eje z: D = D a1 1r. S lo existirÆ flujo a travØs de la superficie lateral del r

ElectrostÆtica cilindro, ya que en las tapas D es perpendicular a dS y su producto escalar serÆ nulo. Por otra parte, en la superficie lateral ambos vectores son paralelos y su producto

escalar equivale al producto de sus m dulos. As : ⋅dS = ∫S Lat.

. s

∫ D⋅dS = ∫

S Lat.

.

D1

D1r ⋅dS

1

Como r es constante en todos los puntos de la superficie lateral, D tambiØn es constante y puede salir de la integral, por lo que:



D1r ⋅dS = D1r ∫S Lat..dS = 2 rhDπ 1r Por otra parte, la carga Q encerrada en el cilindro vale, teniendo en cuenta que la densidad superficial de carga se distribuye uniformemente: S Lat..

Q = v ρ ρsS = Y aplicando la Ley de Gauss:

s

v

2πrhi

2πrhD1r = ρπs 2 rh i ⇒ D1r = ρs rri ⇒ D1 = ρs rri ar Calculemos ahora la contribuci n del conductor 2. Al eliminar el conductor 1, es fÆcil advertir que el vector desplazamiento elØctrico creado por un cilindro hueco como el conductor 2 es nulo en su interior, ya que si aplicamos la Ley de Gauss, la carga libre encerrada por cualquier superficie cerrada contenida en el interior del cilindro es cero y, por tanto, el desplazamiento es nulo: D2 = 0.

As , el desplazamiento elØctrico en el interior del dielØctrico se debe s lo al conductor interior: D= D1 . El dielØctrico es un medio simple, y esto nos permite escribir: 





E = εD = εεD0r ⇒ E= rρεε0srir ar Dado que el campo elØctrico en el dielØctrico es inversamente proporcional a la distancia r al eje del cable, el campo mÆximo se producirÆ para la distancia m nima, es decir, en la superficie del conductor interior (r = ri ): Emax = E(r )i= riρ ρεε εεs0ri r = 0 s r ≤ 5 10⋅ 6 V/m Esto nos permite hallar la siguiente condici n para la densidad de carga, ρs :

ρ εεs ≤ 0 r5 10⋅6 C/m2 La diferencia de potencial entre dos puntos viene dada por:

ElectrostÆtica Vo −Vi = −∫r Crio, E dl⋅ , donde podemos elegir cualquier camino C. Lo mÆs sencillo es integrar a lo largo del camino seæalado en el esquema de la secci n transversal del cable (en la direcci n radial), donde E es paralelo y de sentido opuesto a dl: Vo −Vi = −∫rrio E dl⋅ = ∫rrio E dr⋅= ∫rrio rρεε εε0srir dr = ρ

r (lnr

0s ir

−lnr ) =10 V Despejemos ρs para obtener la condici n que ha de cumplir r : o 104εε ρs = r (ln rio −0 ln r )r i ≤εε0 r 5 10⋅

6

⇒ ln ro − ln ri ≥1 ⇒

ro ≥ e1+ln(0.002) 0.0054 m = 5.4 mm As que el conductor externo debe tener un radio interior menor o igual que 5.4 mm, para asegurar que el campo elØctrico en el aislante no supera el 25% de su rigidez dielØctrica.  Ejercicio 19 y

ρs + + + + + + + + + dielØctrico de permitividad dielØctrica relativa.

ε r

x Q

Se nos pide la capacidad, C, que se obtiene como C = . Calcularemos V a V partir del campo e integrando. Suponiendo que las cargas se distribuyen uniformemente tendremos una densidad superficial en cada placa +ρs y −ρs , donde: Q = ρs ⋅S → ρs

=Q

S

Las condiciones de contorno entre el dielØctrico y las placas vienen dadas por: E1t = E2t = 0 → E1 = E1n ; E2 = E2n

ElectrostÆtica

(D1n − D2n ) = ρs indicando con 1 el dielØctrico y con 2 el metal. Como en los metales no existe campo en su interior:

E2n = E2 = 0

Entonces, como

D1n =ε ⋅ E1n = ε⋅ E1 = ρs

ε=εε 0r

=

E ρs =ρs = Q 1

ε Sε ε0εrS

CÆlculo de V12 : yd

d





y

V12



Q  . S

=0 dy

d

∫ (

ε0 S .

Q 1 y2 1− 2 3d

)

Q ε0 S

d

1 3d 2

∫ 0

3d 2 − y 2

=

= =0 Edy dyεε0 r  = 0 = dy =

=Q

ε0

0

Q

d



0

d

=2 ∫3d 2dy − ∫ y2dy = 3ε0QSd 2  = 3ε0Sd 0

ε0

0

( )− y3

3d 2 ⋅ y

0d

3 0d

ElectrostÆtica ε

Q

Q

= 3 0QSd 2  3d 3 − d33  = 3ε0 Sd 2 9d 3 3− d 3  = 3ε0 Sd 2 d

Q

8

8 Qd

8 3 3  = 9 ε⋅0 Sdd 32 = 9 ε

0

S = V12 

V12 = 98εQd0S (Voltios) Calcularemos ahora la capacidad como:

C=Q= (Faradios Qd

Q = Q ⋅9⋅ε0 S = 9ε0 S ) 8d

8d

0

 Ejercicio 20 Segœn los datos del problema tenemos la siguiente distribuci n:

,

C = 9ε0 S

ElectrostÆtica 



vector polarizaci n (a) Para determinar el campo elØctrico (P), utilizaremos las siguientes relaciones: D=ε0E+ P (ecuaci n 1) E ), el desplazamiento elØctrico ( D ) y el





∇DD==εEρ

(ecuaci n 2) v

(ecuaci n 3)

El enunciado nos dice que el dielØctrico no contiene cargas libres en su interior. Como el problema es unidimensional, la ecuaci n 3 implica que D es constante dentro del condensador: ∂ D

∇D = ∂ y = 0 → D = cte

Por otro lado, como el campo es conservativo:

C

∫E•dl = −∆V , donde

Cy

−∆ V V = 0 ≡ ∈ [0, d]



De la ecuaci n 2 despejamos E y sustituyendo el resultado en la integral anterior: dl = dy D ≡ cte

d

εD

∫ d 00

D

, donde

y

ε ε= +0d

(1+dly /d) =V0 → εD0 d ln(1+ dy )]0d =V0 ⇒V0 = εD0 d ln(2)

ElectrostÆtica ε

D = V0

0

⇒ D



ε

= V0

0



a

y

d

ln(2) d ln(2) 

La expresi n obtenida para D la sustituimos en la ecuaci n 2 y as conseguimos determinar E :

E= εD , con ln(2)V[10+ y / d]ay

D = εV00 d ln(2)y

E= d

 ε ε= 0 + d

Despejando el vector de polarizaci n en la  y E, se tiene que: P= D−ε0 E ecuaci n 1, y sustituyendo las expresiones halladas para D

P = V0ε0 • +y y ⇒ P= dVln(0ε02)•d +y y ay . d ln(2) d b) Calculo de las densidades de polarizaci n.

• Densidad de carga volumØtrica: ρ = −∇P PV

−∇P= − ∂P ≡ − V ε ∂

• Densidad de carga superficial en el dielØctrico: origen, y=0:

(2) en y=d: 

1

ρPS = P•an (1) en el

ElectrostÆtica S

y

y

c) Calculo de la capacidad. Q

C = ∆V Donde ∆V =V0 , y Q es la carga en cada placa, que vendrÆ dada por: Utilizaremos la siguiente f rmula:

Qv = ρS •A Siendo ρS la densidad superficial de cargas libres en las placas, y A la superficie de dichas placas. Por otro lado, sabemos que a partir de la condici n de contorno para el desplazamiento elØctrico entre cada placa y el dielØctrico, se cumple paraρS : D1n − D2n = ρS Como en un metal el campo elØctrico es nulo: D2n =ε0 •E2n = 0 → =ρS D1n = =D D1

Por coincidir la componente normal del campo elØctrico con su modulo. As ,

ρS = V0ε0 Qv = ρS •A = AV0ε0 . d ln(2) d ln(2) (Para la carga de la segunda placa, el resultado seria lo mismo con el signo opuesto, ya que D1n seria el del metal y D2n el del dielØctrico). Una vez calculada la carga, la capacidad del condensador serÆ:

Q

/

)

ε

C = ∆ V = AV0ε0V d ln(2 = dAln ( )02 0

ElectrostÆtica  Ejercicio

21

Expresiones que contienen ρV y ρPv : ∇D=ρ , como D

a)

ρ

llegando a

V

 

∇ P 

= −ρPv

(

)

   = ε0 E + P queda

.

Con

la







0

consideraci n de

 E +

) ρ

 P =

V

que

obtenemos:



∇ε0 E −ρPv =ρV

b)

Pv

0

Ahora bien, segœn el enunciado del problema

ρ = 0, cuya sustituci n en cualquiera de V

las ecuaciones anteriores nos da el resultado de: . ε 0 ∇E = ρ Pv Con lo que demostramos que en un dielØctrico existe densidad volumØtrica de carga ligada en ausencia de carga libre. Ahora obtengamos su valor. Al ser el medio lineal se cumple:

D=εE, expresi n que llevada a las ecuaciones del principio lleva a

( )

que ∇ εE = 0 . 

Desarrollando el operador ∇ , al ser el medio no homogØneo:

ε∇E+ E∇ε= 0, quedando finalmente la expresi n

, que sustituida en

ε∇E=ρPv 0

ElectrostÆtica Ejercicio 22 se obtiene

ε0  − Eε∇ε = ρPv .

Haciendo el cambio ε=ε0εr se llega al resultado final:

ε0  − Eε∇0εεr0εr  = ρPv

cuya simplificaci n es igual al resultado de las soluciones: 

ε0  − Eε∇r εr  =ρPv 



Nos piden la relaci n de la permitividad relativa que permita que el m dulo del campo sea independiente de r. Para ello hallaremos primero el campo elØctrico entre los dos  E  r1

cilindros, suponiendo que en la superficie externa del conductor interior hay una carga superficial -Q, y en la superficie interna L del conductor exterior +Q.. Sabiendo que: D=εEε=ε0εr ⇒ D=ε0εr E Por el postulado fundamental:



S

DdS= QV

Donde S representa la superficie a trazos de la figura (la suma de las tapas mÆs el lateral). En las tapas el vector D es perpendicular al vector dS Y por lo tanto se anulan. Por lo tanto, s lo se calcula el flujo a travØs de la superficie lateral.  dS

 D D

-Q



 dS

r1 Q

∫DdS = D ∫dS = DS

lateral

= D2πrL

r

ElectrostÆtica  Ejercicio

23

Slateral

Slateral

D2πrL = Qv

D =ε0εr E ⇒ E = 2πrLQεv 0εr

Para que E sea independiente del radio, el dielØctrico ha de ser sea inhomogØneo, con: K εr = r

εr ∝ 1r →

Q



Con lo que el campo queda: E= 2πKL vε0εr a

r

Ahora nos preguntan por la densidad volumØtrica de carga.

( (

) )

ρpv = −∇P= −∇ εo ⋅ εr − 1 ⋅ E = −∇ k −r r ⋅ 2πQKL ⋅ ar  = =− =

Q⋅ ∇ k − r ⋅ ar  = −

Q⋅ 1 ⋅ ∂ r ⋅ k − r 

Q 2πKL

P

r



2πKL r ∂z 

r

2πKLr

Un electrete es un dielØctrico que se conserva polarizado indefinidamente, despuØs de ser sometido a un campo elØctrico intenso. La polarizaci n da lugar a una carga positiva neta en una de las caras y negativa en la otra.

Teniendo en cuenta que



P

= P*uz

y que la carga por unidad de Ærea sobre la superficie de un

ElectrostÆtica Ejercicio 24 material polarizado es igual a la componente de la polarizaci n que estÆ en la direcci n de la normal a la superficie del material, podemos considerar que P =σp ( en la cara superior). En el interior del electrete, al ser la polarizaci n uniforme, no existe densidad volumØtrica de carga de polarizaci n. As pues, para resolver el problema vamos a calcular primeramente el campo debido a un disco con densidad superficial σp en los puntos de su eje.

  (r −r ’)σ p dS E=  3 S 4πε0 ∫ r −r ’ 1

r





= zu

  r ’= r’u r

z

dS = r’*dϕ*dr’

(

  2 2 r − r ’ = z + r’

)

12

El campo buscado solamente tiene componente z. R

1

2

R

σ

Ez 4πε0 ∫0πdϕ∫0 (z 2p z+*rr’2’dr)32’ = σ2εp0 − (z 2 +zr’2 )12  0 = =

σ2εpo 1− (z 2 +zR2 )12 

Nota : La integral se ha resuelto mediante el cambio de variable siguiente: z 2 + r’2 = t 2

El resultado es : E= Ezuz

(por encima del disco)

 

E = −Ezu

z

(por debajo del disco)

ElectrostÆtica 

Ahora vamos a calcular el campo E en un punto del eje fuera del electrete. Para ello nos damos cuenta de que el campo buscado es igual a la suma de los campos creados por dos discos con densidades superficiales σp y −σp situados en z = t y z = 0, respectivamente.

E= σp 1−

((z −t)z −+tR ) 2

2

12



−1− (z 2 +zR 2 )12 uz o

2ε0

 Si operamos en la expresi n anterior, cambiandoσp por P, obtenemos:

(

Eo = 2−εΡ0  (z −t)z2−+tR 2

) − (z 12

2

+zR2 )12 uz



El desplazamiento elØctrico =ε0 E+ P . D se calcula, de forma general, con la f rmula: D 

Para calcular D 

o

tenemos que tener en cuenta que en la zona en que hay vac o el

P

vector D o =ε 0E se anula. El resultado es por tanto: o Ahora vamos a calcular el campo E en un punto del eje en el interior del electrete. Operando de manera anÆloga al caso anterior, teniendo en cuenta que ahora los campos son aditivos, se obtiene: σ  

ElectrostÆtica

Ei =

2εp0 −1−

uz = ε

=

2 p − 2 + +zR2 0

) − (z 12

2

2

( t−z) +( 1

2

2

12

 −1− (z 2 +zR2 )12

+zR2 )12 uz = 2−εP0 2 −

((t − zt) − 2

+zR2 )12 uz σ 

(t − z)2 + R2



((t − z(t) −+zR) )

z

2

Como el problema nos dice que el electrete es delgado podemos hacer la siguiente aproximaci n: R >> t  R >> z y R >>(t-z)  R2 >> z2 y R2 >> (t-z)2 Teniendo esto en cuenta, obtenemos lo siguiente :

)

Ei = 2−εP0 2 − t −Rz − Rz u z = 2−εP0 2 − R1 (t − z + z u z = 







2−εP0 2 − Rt u z = 2−εP0  2RR−t u





z



El desplazamiento elØctrico D i se calcula como: D i =ε0 E i + P



)

D  2R −t  + P= P − 2R + t + 2R  = Pt

(

=ε0 2−εP0  R 



2R

 2R i

Ejercicio 24 Condiciones de contorno: * Ein,t = Eout,t * D Din n, −

out n,

= ρs = 0⇒D Din n, =

out n,

El problema no nos indica que existe carga libre en la superficie del dielØctrico, por lo tanto, ρs = 0. TambiØn nos dice que el dielØctrico es homogØneo, as que,

ElectrostÆtica 

asumiendo que tambiØn es is tropo, podemos afirmar que el medio es simple, por lo que: D=ε⋅ E Para poder demostrar las condiciones de contorno, tenemos que calcular el campo tanto en el exterior y en el interior de la esfera. En coordenadas esfØricas se tiene que: →

E=−∇V =−∂∂VRa→r −R1 ⋅∂∂θVa→θ

ya que no existe dependencia de los potenciales con φ. Calculemos entonces el campo en el interior:

Vin

Ein = 3E0 cosθa R+ 3E0(−senθ) aθ (εr + 2) (εr + 2) Ahora calculemos el campo en el exterior de la esfera:

vout = −E0 cosθ+ ER0a2 3 ⋅εεrr +−12

R

R3 εr + 2 ∂

Vout

E0a εr −1 = E0 R ⋅ senθ + R 2 ⋅ ⋅ (−senθ ) ∂θ εr +2

3

3

Eout = Eo cosθ1+ 2Ra3 ⋅εεrr +−12aR − E0senθ1− Ra33 ⋅εεrr + −12aθ

Pero como estamos en la superficie (R = a), tenemos que el campo elØctrico en el exterior de ella es:

ElectrostÆtica

Eout (R = a) =

aR +



Ahora que hemos calculado el campo, comprobamos que se cumplen las condiciones de contorno: * Ein,t = Eout,t Ein,t = Eout,t = por lo que se cumple que las componentes tangenciales del campo son iguales. * Din n, =Dout n, Din,n =ε0εr E1n = Dout,n =ε0 E2n = Con esto se demuestra que las condiciones de contorno en la superficie de la esfera se cumplen. Aclaraciones: - La componente tangencial del campo depende, en esfØricas, de los parÆmetros θ y φ. Al ser la componente φ nula, la œnica componente que actœa es θ. - La componente normal del campo es la que depende del parÆmetro R. - En la expresi n de la componente normal exterior, el parÆmetro εr , puesto que en el vac o su valor es 1, se corresponde al dielØctrico. Ejercicio 25 En general, la expresi n de la energ a de un sistema discreto de cargas es la siguiente: W = 2∑i=n1 qiVi , donde Vi = potencial en la posici n i debido a las demÆs cargas 1

ElectrostÆtica 

Para saber cual de las dos situaciones es mÆs estable hallamos la energ a en las dos situaciones, y aquella donde la energ a es menor es la mÆs estable. Situaci n original: 1 d

2 d

• q

3

4

• q

• -q

d • -q

V1 = 4πε1 0 −dq + 2qd − 3qd  = 24πε5q0d

V2 = 4πε1 0 dq + dq − 2qd  = 8πε3q0d

V3 = 4πε1 0 2qd − dq − dq = 8πε−30qd





V4 = 4πε1 0 3qd − 2qd + dq  = 24πε5q0d

WOriginal = 12 ∑i=n1 qiVi = −7q12πεq 0d Situaci n tras el desplazamiento:

d

1

2

d/2

3d/2



• q

3 •

-q

4 •

q

-q

ElectrostÆtica V1 = 4πε1 −dq + 3dq 2 − 3qd  = 6πε− q0d 0  V2 = 4πε1 0 dq + dq2 − 2qd  = 8πε5q0d

V3 = 4πε1 0 3dq 2 − dq2 − 3dq 2 = 2πε− q0d

V4 = 4πε1 0 3qd − 2qd + 3dq 2 = 8πεq0d

Wdesplazado = 12 ∑i=n1 qiVi = −17q 24πεq 0d Wdesplazado es menor que Woriginal, por tanto, el sistema es mÆs estable tras el desplazamiento.

ElectrostÆtica 

ElectrostÆtica  Ejercicio

26

R4

R3

R R1 R2

Soluci n: En el problema 4 obtuvimos los potenciales en las distintas regiones del sistema, que son: *Para r > R 4 : V = 0 *Para R 4 > r > R 3 : V = 0 Q

* Para R 3 > r > R 2 : V = 2πε

*Para R 2 > r > R1 : V

=Q

+ Q ’

+ Q ’

2πε

0

 R

R

3

2





0

L ⋅ Ln

R 

 r3 

 R

Q







L ⋅ Ln

*Para R1 > r > R : V = 2πε

L ⋅ Ln

0



R3  2



R

R



Q’

 R12 r3  + 2πε

0

L ⋅ Ln

*Para R > r : V = 2πεQ0 L ⋅ Ln  RR12RR3  + 2πεQ’0 L ⋅ Ln  RR23   El sistema es anÆlogo a dos condensadores conectados en serie. La energ a total serÆ igual a la suma de la energ a de cada uno de estos condensadores:

ElectrostÆtica Ejercicio 27

ω1 = 12Q⋅[V( )R -V( )R1 ]= 4πεQ 20 L ⋅Ln  RR1 

 RR32  ω2 = 2(Q

12

+Q’)⋅[V(R2 )-V(R3 )]= (Q4πε+Q0 L’) ⋅Ln Por tanto, la energ a total es:

ωT =ω1 +ω2 = 4πε1 0 Q 2 ⋅Ln  RR1 +(Q +Q’)2 ⋅ln  RR32 



Para Realizar el cÆlculo de la Energ a electrostÆtica almacenada en el sistema del problema 7, necesitaremos los siguientes valores calculados en el mismo:

Q 

V1 = 4πε

Q 

V2 = 4πε

0 R3

3Q

V4 = 4πε

1

3

1

1

3





 r − R 2 + R 3  

0

3Q

V3 = 4πε

1

 R 1 − R 2 + R 3 

0

r

0

r < R1

R1 ≤ r < R2

R2 ≤ r ≤ R3

r > R3

Para calcular la energ a almacenada en el sistema, nos basamos en que este es anÆlogo a 2 condensadores en serie: uno entre los dos conductores, y otro entre el conductor externo y el infinito . Por lo tanto, aplicamos en cada uno de ellos la f rmula de la energ a para un condensador:

ElectrostÆtica  Ejercicio

28

ω= Q∆V ω1 = 1 Q[V(R1 ) −V(R2 )]= 1 Q4πεQ 0  R11 − R12 + R33  − 4πε3Q0 R3  = 8πεQ 20  R11 − R12  2 2

ω2 = 12 Q[V(R1 ) −V(R2 )]= 123Q4πε3Q0 R3  = 8πε9Q0 2R3

Sumando las energ as resultantes obtenemos la energ a Total: Q 2  1

1

9

ω=ω1 +ω2 = 8πε0  R1 − R2 + R3 

En primer lugar debemos de darnos cuenta que para calcular el trabajo total tendremos que calcular el trabajo para traer la carga Q desde el infinito y luego calcular el trabajo para traer la carga Q + Q/2 desde el infinito. Una vez obtenidas estas dos cantidades haremos la resta de WQ+Q/2 - WQ. Para calcular dichos trabajos usaremos la f rmula: →→

W=

D Edv

Como podemos observar en la f rmula del trabajo necesitamos saber el valor del campo para poder responder al problema. CÆlculo del campo:

ElectrostÆtica Ejercicio 29 Observamos como hay tres regiones: una con R R2. Calculemos el campo en cada una de las regiones. Como el problema tiene simetr a esfØrica aplicamos el Teorema de Gauss para el cÆlculo del campo. Regi n con R > R2 Φ = ∫ E→ ds→ =

E

∫ds = E ⋅4πR

E ⋅4πR 2 = Q /ε2o+ Q

E 8πR εo Regi n R1< R < R2 Φ = ∫ E→ ds→ =

∫ds = E ⋅4πR

E

2

E ⋅4πR 2 =

E = 4πRQ2εo

Q

εo

2

ElectrostÆtica 

Regi n de RR1.

WQ = 12 ∞∫D→ E→dv 2 ∞

2 R14πR ε0 

8πεQ02R1

- Energ a de Q y Q/2 WQ+Q / 2 = ε20 RR∫124πεQ0 R 2 dv

2 R28πε0 R 2 dv = 8πεQ02R1 −

8πεQ02R2 + 2πε9R01R2 2 Una vez obtenidos las dos energ as las restamos y obtenemos el trabajo realizado para acumular sobre la superficie esfØrica de radio R2 una carga de Q/2.

∆W = WQ+Q / 2 −WQ = 32πε5Q02R2

ElectrostÆtica

Ejercicio 29 a) Campo en funci n de la distancia al centro:

 En 0 ≤ r ≤ R1 Como la densidad de carga, ρ, es uniforme en la esfera, de volumen V: Q = ρ⋅V ⇒ Aplicando el teorema Gauss:

  Q   E ⋅ ds = v ⇒ E ⋅ ds =ρ ⋅ ∫ ∫ ε0 ε0 V

=

, donde, resolviendo la integral:

Q

=

Q

E ⋅ 4πr 2 4/3πε R 3 ⋅ 4/3πr 3 , quedando E1 4πε 0

0 R13

⋅ rU→r

1

 En R1 ≤ r ≤ R2

∫E⋅ds= εQ ⇒ E ⋅ 4πr v

2

= εQ0 ⇒ E2 = 4πεQ0 r 2 U→r

 En r E3 = 0, ya que la carga total encerrada en las superficies esfØricas para las que r ≥ R2 es cero, con lo cual si no hay carga tampoco habrÆ campo. b) Energ a electrostÆtica del sistema: Como solo existe campo en la regi n 1 y en la regi n 2, la correspondiente energ a → electrostÆtica, cuya expresi n depende directamente del E , vendrÆ definida por los l mites de integraci n correspondiente a las separaciones entre las regiones 1 y 2. Por otra parte, la expresi n de la energ a electrostÆtica creada por un sistema viene dada por:

We

dv

ElectrostÆtica 

En donde calculando la energ a de nuestro sistema obtenemos:

We = 12ε0 R∫1E 2 ⋅4πr 2 dr + R∫12E2 2 ⋅4πr 2 dr = 12ε0 R∫01

(

)

Q 42πε⋅r 02 R⋅ 143π r2 2 dr + 5R∫R11 (Q4πε2 ⋅ 04rπ2r)22 dr = 1

0



R

12ε0 82Q 22  R∫01 Rr146 dr + 5R∫R11 rr 24 dr = 8πεQ 20  5⋅rR5 16 0R1 + − 1r 5RR11  = 8πεQ 20 R πε0 

c) Si quitamos la mitad de la carga Q de la capa esfØrica, ¿CuÆl serÆ la variaci n de la energ a electrostÆtica del sistema? Al quitar la carga Q/2, la carga resultante serÆ -Q/2, cosa que œnicamente afecta a E3, que deja de ser nulo. Ahora: v

⇒E

v





⇒ E3 = U r 4πε0 r Por tanto, como consecuencia de que el campo elØctrico en la regi n 3 no es 0, la variaci n de energ a electrostÆtica vendrÆ determinada por:

We = 12ε0 5∞∫R1 (8πεQ02r 2 )2 ⋅ 4πr 2 dr = 32Qπε2  5∞R1 ⇒ ∆We = 160Qπε2 0 R

0

− 1r

ElectrostÆtica

Ejercicio 30 Tenemos un condensador plano de superficie S y espesor d (separaci n de las placas). El condensador se carga con una diferencia de potencial de V 0. Nos piden hallar la densidad de energ a electroestÆtica. Para hallarla es necesario conocer el campo entre las placas: + + + + + + + + + +

d

Por ser el campo uniforme, perpendicular a las placas:

-

-

-

E

E = Vo

---

-

d

-

--

Hallamos primero la densidad volumØtrica de carga, Como D=ε0E:

DE.

ElectrostÆtica 

dWdveA = 12ε0E2 = ε20 Vd022 Multiplicando por el volumen total:Volumen = S d• , ε

V

We = 20 S d02 A

En la segunda situaci n introducimos una lamina metÆlica de espesor d/2 entre las dos placas del condensador, tras haber desconectado la fuente de tensi n. Debemos hallar el campo en las regiones 1 y 2. d/4 d/2 d/4 +++ 1

1 2

Como las cargas en las placas del condensador no han

--

variado,

+-

el campo debido a las placas, entre Østas y el metal

(1), +

++

V0

-

--

. Obviamente, en el interior del

+ -

permanece

constante: E1 = d el campo es nulo: E2 = 0. Por tanto, la densidad en (1), serÆ: + - metal

+-

volumØtrica de energ a

dWdveB = 12ε0E2 = ε20 Vd022

y en (2) serÆ cero. El volumen ahora serÆ: Volumen energ a: ε

=  d + d  4  4 S . Y la

V

We = 40 S d02 B

Por œltimo, la diferencia de energ a entre ambas situaciones es : ∆W =ε0 Vd02 S 12 − 14  =ε40 Vd02 S

La pØrdida de energ a se traduce en el trabajo realizado por las placas al introducir la lÆmina (o como el trabajo que ser a necesario para sacar de nuevo la lÆmina metÆlica).

ElectrostÆtica  Ejercicio 31

+ + + + + +

 E1

 E2

 E1

w

-

d V a) Capacidad del condensador: C Q = ∫ρ ρSdS =

S

⋅A

Al tratarse de un condensador s lo existirÆ campo entre las placas del mismo. Por tanto:

V = ∫Edl= E1 ⋅(d −W) + E2 ⋅W Por Gauss (empelando como superficie gaussiana un cilindro en una de las placas del condensador): dS E A1 E1

1

0

D1

Por otro lado, sabemos que una condici n de contorno es: D1n − D2n = ρS . Como en la superficie del dielØctrico no hay carga libre: ρS = 0, y teniendo en cuenta que el vector desplazamiento elØctrico en este problema s lo se mueve en la direcci n perpendicular a las placas (componente normal) tenemos que: D1 = D2 ;ε0 ⋅ D1 =ε ε0 ⋅ r ⋅ D E2 ; 2 = εEr1 Si sustituimos en el potencial: V = E1 d −W + ε εWr  = Er1 εr ⋅(d −W ) +W 

ElectrostÆtica

Con lo que capacidad serÆ: C=Q=

ρS ⋅ A

= ε εr ⋅ 0 ⋅ A

= 321pF

E d W1 ⋅( − ) + E W2 ⋅ εr ⋅(d W− ) +W

V

b) Carga del condensador: sabemos que, Q=

ρ S

⋅A

V = εEr1 εr ⋅(d W W−) + ⇒ E1 = εr ⋅(d W WV−⋅εr ) +

ρ=E

⋅ε = 1.61 10⋅

−5

Q c) Energ a almacenada: Partiendo de la f rmula general de la energ a, podemos deducir una expresi n para la energ a de un condensador. We 2 ∞ D E dv D E dv Sabiendo que: dv = A d⋅ l D = ρS Sustituyendo se encuentre que: We

AD E dl A D

We

AV

V

E dl

ADV

CV

J

d) Calculamos los vectores campo, desplazamiento elØctrico y vector de polarizaci n: D1 = D2 = ε0 ⋅ E1 = 161 10. ⋅

−5

E1 = εr ⋅(dV−⋅Wεr) + W = 181818182. 

E

 

E2 = εr1 = 45454545.a x V m

ax Cm2

ElectrostÆtica 







 C

D1 = ε0 ⋅ E1 + P P1; 1 = D1 − ε0 ⋅ E1 = 0 m2 D2 = ε0 ⋅ E2 + P P2 ; 2 = D2 − ε0 ⋅ E2 = 12 10. ⋅

−5

ax Cm2

La representacin de estos vectores serÆ:



Ejercicio 32 a) Hallar ρ0 en funci n de Q y a. Ra

∫ Dds = Q = Q

2

v

Q E2 = ε

∫εEds =ε π E4 R 4πR o

o

2

ElectrostÆtica

b) Hallar la energ a electrostÆtica del sistema:

W = 12



DEdv = 12 ∫0aεE1 ⋅ E dv1 +



ε E2 ⋅ E dv2  = 12 ε

a∞ o

π∫0a E12 4 R dR2 +ε0 ∫a∞ E22 4πR dR2  = sustituyendo valores y calculando las integrales

= Q 2 ⋅εo + 2ε 24πa ε⋅εo



Ejercicio 33

Dividiremos el estudio de este problema en dos partes bien diferenciadas. Nos centraremos en primer lugar en calcular la distribuci n de potencial en la zona 0 < φ C(α- 2π) = V0 => C = −

V0

(2π−α) Por tanto: D = (2π2π−α)V0 La distribuci n del potencial serÆ: 2π

V =

φ+ (2π −α)V0

φ

V = 22ππ−−α V0  Ejercicio

34

Qs = Carga inducida por Q en la superficie del conductor.

ElectrostÆtica

P

S, ρs

a P2

Q

P1

D V=0

Para resolver este problema por el mØtodo de las imÆgenes, se debe sustituir el conductor por una distribuci n de carga para hacerlo mÆs sencillo, pero que siga manteniendo las condiciones de contorno que hab a con el conductor. En este caso, en la superficie el potencial debe ser 0 porque estÆ conectado a tierra. Utilizaremos una carga puntual imagen que situaremos dentro de la esfera conductora. A continuaci n se calculara el campo En (campo elØctrico normal a la superficie) ya que sabemos que:

Qs = ∫ρsds ; ρs =ε0 E



n

Q= carga puntual imagen

P3

r1 Q

P2

r2 Q

θ b

P1

D V=0

Una vez sustituimos el conductor por la carga imagen, comprobamos que las condiciones de contorno no han variado. En este caso debemos comprobar que el potencial en cualquier punto donde se encontraba la superficie del conductor es 0.

ElectrostÆtica

Primero lo comprobamos para P1 y P2: V(

)

V(

)

Resolviendo este sistema de ecuaciones hallamos:

b=

a2

Q’= −

D

aQ

D

Para estos dos puntos se cumplen las condiciones de contorno, pero falta comprobarlo para cualquier punto de la superficie de la esfera. Sea P3 un punto cualquiera de la superficie de la esfera conductora. Por trigonometr a:

V( )P3 = 4πε

1

0



Q

Q



⋅  r1 + r2’ 

r1 = D 2 + a 2 − 2aD cos( 180 −θ)

r2 =

a 2 + b 2 − 2abcos(180 −θ)

cos(180 −θ) = cosθ 



V( )P3 = 4πε1 0 ⋅ D 2 + a 2 Q− 2aDcosθ +a 2 + a−42aQ− 2DaD3 cosθ⇒V( )P3 = 0 

D

Esto implica que Q esta bien situada. Una vez comprobado que se cumplen las condiciones de contorno, debemos hallar V en un punto cualquiera.

ElectrostÆtica P

r3

P2

Q

r

r4 Q

θ

P1

b

V=0

V( )P = 4πε1⋅Qr3 + Qr4’  = 4πε1 0 ⋅Qr3 − aQr4 D 0

r3 = r 3 + D 2 + 2rDcosθ

r4 = r 2 + Da42 − 2r aD2 cosθ

Sabemos que el campo en la superficie serÆ: E





= E r (r = a)

n

aQ ⋅

 ⋅θ

En general: Er = − ∂∂Vr = 4πε1⋅Q(r + Dr 3 cos ) + D r +r4a3D2 cosθ 0



3





Con lo que:

En = Er (r = a) = −

Q 4

a ⋅ (D 2 +

Qs = ∫ sρsds;

D 2 −a 2 a 2aD cos ) 3 2 +

θ

ds = a 2 senθdθdφ;

S

πε0

ElectrostÆtica La carga que aparece en la superficie coincide con la carga imagen Q .

ELECTRICIDAD Y MAGNETISMO. Magnetostática-Fundamentos 1) Mediante la ley de Ampere calcular el vector densidad de flujo magnético que existe, cuando circula una corriente i, a) en el interior de un solenoide de N espiras y longitud L (L suficientemente grande). b) en el interior de una bobina toroidal de N espiras y radio externo medio, a . c) por un hilo conductor infinito.

2) Dos conductores rectilíneos, paralelos y muy largos, separados una distancia 2d, transportan corrientes de igual intensidad pero con sentidos contrarios, como se indica en la figura. Calcular: a) El vector densidad de flujo magnético B en un punto genérico del eje OX, su valor máximo y el punto donde se localiza. Representar gráficamente B(x). b) El vector densidad de flujo magnético B en un punto genérico del eje OY. Representar B(y). Y I

. 2d x

3) Los electrones de un haz cilíndrico de radio a, se mueven con velocidad, v, constante y dirigida a lo largo del eje, de forma que se mantiene una distribución uniforme de n electrones por m 3. Siendo a=1 mm, v=2 ·107msg-1 y n=5·1010 electrones·m-3, determinar: a) La densidad espacial de carga, la densidad de corriente eléctrica y la intensidad de la corriente. b) El campo eléctrico en la superficie del haz. c) El vector densidad de flujo magnético en la superficie del haz. d) Las fuerzas de origen eléctrico y magnético que actúan sobre un electrón situado en la superficie del haz, y el cociente entre ambas.

4) Un cable delgado, que transporta una corriente I, está doblado en ángulo recto tal y como indica la figura. Calcular B a lo largo del eje OX, suponiendo que el cable es infinitamente largo en ambas direcciones.

I

O

X

ELECTRICIDAD Y MAGNETISMO. Magnetostática-Fundamentos

5) Calcular el vector densidad de flujo magnético en el eje de una espira circular de radio a, por la que circula una corriente I. 6) Encuentre la densidad de flujo magnético, B, en el centro de una espira cuadrada plana de lados w, por la que circula una corriente I. 7) Por un filamento conductor con forma de triángulo equilátero de lado a, fluye una corriente constante I. Calcular la intensidad del campo magnético en el centro del triángulo. 8) Disponemos de un conductor cuya forma es la indicada en la figura. Este conductor se prolonga hasta y=-∞ y z=+∞. Por dicho conductor circula una corriente I en el sentido de la figura. Calcular mediante la ley de Biot-Savart la inducción magnética en el punto P de coordenadas (0, 0,-b).

a

P

9)

Una corriente constante con densidad superficial

KO

a z

, fluye en el plano y = 0. Calcular la densidad de

flujo magnético que se genera a ambos lados de dicho plano.

MAGNETOST`TICA.- FUNDAMENTOS Problema 1 a.- En el interior de un solenoide de N espiras y longitud L (L suficientemente grande)

Hip tesis: Suponemos que B = 0 en el exterior del solenoide B1

B

TambiØn suponemos que B serÆ constante a lo largo del interior. c

db

a

B dl Bl porque, o bien B es

perpendicular a dl (en puntos

interiores del bc

solenoide), o el campo es cero (en puntos exteriores al solenoide) porque el campo es cero (puntos exteriores al solenoide) cd

estar amos en un caso similar al de ∫ Bdl bc

As que la integral a lo largo de la trayectoria cerrada vale: Bl Y aplicando ahora la ley de Ampere: Inl donde nl es el numero de espiras que existe en nuestra secci n de longitud l, ya que n es la densidad de espiras (numero de espiras por unidad de longitud). Con lo que la densidad de flujo magnØtico queda: B = µ0 In Sustituyendo n = solenoide, se tiene que:

N

, siendo N el nœmero total de espiras y L la longitud del L B = µ0

I

N

L

b.- En el interior de una bobina toroidal de N espiras y radio externo medio a

Para puntos del interior de la bobina, Ri < r < Re , B es tangente a circunferencias concØntricas y su m dulo s lo depende de la distancia al centro, r. As , tomando una trayectoria circular con el toroide: .

Ley de Ampere nos dice que:

NI , con lo que,

µNI

B2πr = µ0 NI ⇒ B

Suponiendo el toroide es muy estrecho, r ≈ Ri ≈ Re ≈ a , siendo a el radio medio, se tiene que: B =

µ0 NI

2πa c.- Por un hilo conductor infinito I y como B es paralelo a dl

∫ Bdl = µ I ⇒ B∫dl = µ I ⇒ B2πr = µ I 0

0

Teniendo en cuenta el carÆcter vectorial:

0

B=

µ0I

2πr Problema 2 a)



y

x

Para resolver este problema utilizaremos la ley de Biot Savart y el principio de superposici n. Examinaremos por separado la contribuci n de cada corriente, y las sumaremos. La ley de Biot-Savart nos dice :



B 

α

 B1 

R

d 90º x

α

R

R

 

y

2a 

90º R

  B1 + B2 

α α

x

 B2

µI dl x

En primer lugar se harÆ la representaci n de las contribuciones de los dos conductores en un punto genØrico del eje x. Como vemos, la componente Y de la suma de las contribuciones se anula, y s lo nos quedarÆ componente en X. Las igualdades entre los Ængulos se deben a que el vector aR y B han de ser perpendiculares, por las propiedades del producto vectorial. Sabemos que la densidad de flujo magnØtico para un hilo infinito por el que circula una corriente I vale

µo I : 2πR donde R es la distancia entre el hilo y el punto donde queremos calcular el campo. En nuestro caso:

B = B 1 + B 2 Como s lo queda la componente en el eje X:

B = (B1 ⋅cosα+ B2 cosα)a x cosα= d R Y teniendo en cuenta que las dos corrientes, y por tanto el m dulo de la densidad de flujo magnØtico debido a ellas, son iguales: B

=µπ0RI2d

R = x2 + d2 B =π(xµ20+Idd2) El punto mÆximo se encontrara donde el divisor sea mÆs pequeæo, es decir, donde x= 0:

Bm áx= Representaci n de B(x) 

b)

µId

Haremos una nueva representaci n para ver las contribuciones de las dos corrientes. En este apartado tambiØn utilizaremos el principio de superposici n. Las direcciones de B1 y B2 las hemos obtenido utilizando la regla de la mano derecha. Utilizando la ley de Biot-Savart al igual que en el apartado anterior, obtenemos :

B1

B2 y-d

y y+d

d

x d

Por superposici n :

B 1 = 2π(µy0I− d) − a x

B2 = 2π(µy0+I d) a x

B = B 1 + B 2 =π(dµ20I+dy2)

| B|

=

µId

| Esta f rmula es vÆlida para cualquier punto del eje y. A continuaci n hacemos la composici n de todas las componentes de B en cualquier punto del eje y.

y B1

B2

B1

B2

B2

x

B1

Representacin de B(y)

Problema 3 a) La densidad espacial de carga serÆ constante y uniforme. Su valor es C/m3 La densidad de corriente o cantidad de electricidad que pasa por la unidad de superficie, normal a l direcci n de propagaci n, en la unidad de tiempo, serÆ la contenida en un

cilindro de secci n recta unidad y longitud v, cuyo valor es j = ρ = −v nev = −8 10⋅ ⋅ ⋅2 10 =0,167 /A m2 v ea

−9

La intensidad de la corriente serÆ I = jS = −nev aπ 2=0,16⋅π⋅10 =0,5-6µA b) Aplicando Gauss a una superficie cil ndrica de radio r y longitud l, debe verificarse

∫Eds =



ε Como E y ds son paralelos entre s ocurre

∫Eds = ∫ Eds = E∫ ds = E2πrl Y por otro lado Q =πa l2 ρ = −πa2 lne Con lo cual es E2πrl = −πa2 lne ε° De donde a ne2 E= 2rε° Como nos piden el campo elØctrico en la superficie, hacemos r=a, as ne 8 10⋅ −9 ⋅10−3 E= a == 0,45 V/m 2ε° c) Aplicando la ley de AmpŁre a una circunferencia secci n recta del haz, debe verificarse ∫Bdl = µ°I

r a

 dl   B   

Con esto, teniendo en cuenta que B y

dl

son paralelos, y considerando el radio r de la

circunferencia igual al del cilindro para simplificar, tenemos,

∫Bdl = ∫

Bdl

∫ dl = 2πaB

=B

Con lo cual, teniendo en cuenta el valor de I ya hallado, la ley de AmpŁre se expresa 2πaB = −µ°nev aπ 2 De donde µ°

10−3 =100,531 pT B = − ρva = 2π•10 •2•10 • 2 −9

7

d) La fuerza de origen elØctrico sobre un electr n de la superficie del haz es  B 

a  Fe 

 E

 Fm

La fuerza elØctrica serÆ de sentido contrario al del campo elØctrico, pues es entre dos cargas del mismo signo. F e = -eE= −1,6•10−19•0,45 = −7,2•10−20ur N Y la fuerza de origen magnØtico, que se puede hallar mediante la regla de la mano izquierda, teniendo en cuenta el sentido de la velocidad de los electrones y que la

carga es negativa, 3,22•10−22u N

 r



F m = −e v(

  −19 7 −12 = ×Β =) evBu r =1,6•10 •2•10 •100,531•10

pues B y v son perpendiculares entre s , siendo el cociente de sus

m dulos Fm ° ° 2 = 4,4•10−3 = ε µ v Fe Problema 4 y

I

x

z Mediante la f rmula de Biot-Savart, y utilizando superposici n, hallamos la densidad de flujo magnØtico descomponiendo el circuito en los tramos siguientes:1) Corriente rectil nea sobre la parte positiva del eje Y. 2) Corriente rectil nea sobre la parte negativa del eje X. Siendo entonces: BT=B1+B2

B = µ4π°I ∫c dlR×2aR

1) Para el tramo de hilo vertical tenemos: y I  dl  

 aR R

Ө α

x

Observando la figura podemos establecer las siguientes ecuaciones: dl×aR = −dy sen a• θz x

cosα = R x R= cosα y



senα = Ρ → y=Rsenα = xtgα → dy = x

cos2 α

Quedando entonces

B1

xd α   cos α cos 2  π2 µ° I a z cos αdy µ° I a z µ° I a z α 2 = 4 ∫ 4π ∫ R 2 = 4π ∫ x πx 0

µ

= cosαdα =

2

4πx cos α

2) En este tramo dl





es paralelo a a R , por tanto dl





 

xa R = 0, as B 2 = 0 .

y

I

dl

aR

x

Entonces, la densidad de flujo total serÆ igual a la densidad de flujo magnØtico originada por el tramo vertical. BT = µ°I az 4πx

°

I az

Problema 5 Si partimos del siguiente dibujo: db dl

ar

α

B

r

I

db α

0 a

z r

c

α

B

α

B

Z

db

ar dl db

Para calcular el campo B en el eje de la esfera aplicamos la ley de Bio Savart.

B=

oI

db =

/4π

∫ dl x ar/ r

2

( o.I / 4π) . dl x ar / r2

donde : r es la distancia del punto fuente (donde circula la corriente) al punto campo (donde qeremos hallar el campo). ar es el vector unitario cuya direcci n va del punto fuente al punto campo.

La componente vertical se anula, pues por la simetr a del problema s lo queda componente horizontal, en el eje Z. Si miramos el dibujo de frente podremos observar que dl y ar son siempre perpendiculares entre s . db dl

ar I

ar db

dl

Hablamos entonces de una densidad de flujo efectiva que valdrÆ: db efec = | db| .cos α .az

;

siendo cosα = a/r

Como dl y ar son perpendiculares, entonces : | db | = ( o .I / 4π.) . dl .sen90” / r2 B=

∫ db = ∫ db efec = ∫ ( o.I.a / 4π r

3

) az . dl , la cte sale fuera de la integral

cte B = ( oIa / 4π r3 ) ∫ dl siendo ∫ dl la longitud de la circunferencia que vale 2πa. Entonces sustituyendo ese valor y simplificando obtenemos: B = o.I. / 2 r3

Como r = √(a2 + z2 ) entonces el resultado final es: B = [ o.I.a2 / 2 (a2 + z2)3/2 ].az

Nota: En el centro de la espira (z=0) , el campo magnØtico vale

B = ( oI / 2a) az Problema 6

Haciendo un anÆlisis previo, en la Figura 1, del sentido que lleva la corriente en los lados del cuadrado, vemos que cada uno realiza un aporte a la densidad de flujo magnØtico total en el mismo sentido, el eje z positivo. Por esta raz n el problema lo podemos estudiar de una manera mÆs simplificada, como la densidad de flujo magnØtico debida a un solo lado, representado en la Figura 2, y obtener el resultado final sin mÆs que multiplicar por 4.

Calculemos, pues, la densidad de flujo producida por un hilo finito de longitud w, a una distancia w/2 de su centro. En la Figura 2 hemos hecho un desplazamiento del origen de coordenadas, y , que inicialmente hab amos fijado en el centro del cuadrado de la Figura 1. De esta manera simplificamos los l mites de integraci n que posteriormente aparecerÆn. Tenemos que utilizar la Ley de Biot-Savart, que expresa la densidad de flujo magnØtico debido a un hilo conductor por el que circula una corriente I. lxaR B= µ4π0 I ∫c d

Biot-Savart:

R Particularizando los tØrminos de la ecuaci n para nuestro caso,

R = (y’) 2 + (w/2) 2









dlxa R =| d l || a R | sen(α)a Z = dy’.

w/2 

a

R

R queda finalmente: µI B = 4π0 ∫−+ww//22w2 ((y’)2 +dy(w’/ 2)2 )3/2 aZ =µ4π0I w2(y’)2 +y’(/w2)2 −+ww//22 aZ = 22 µwπ0I aZ Por lo que la densidad de flujo de la espira cuadrada vale:

BTotal = 2 2 µ0I aZ wπ

Problema 7 Por un filamento conductor con forma de triÆngulo equilÆtero de lado a, fluye una corriente constante I. Calcular la intensidad del campo magnØtico en el centro del triÆngulo.

Para hallar

B

B

 usamos el teorema de superposici n. Para ello hallaremos 

en uno de los lados y despuØs multiplicaremos por tres.



dl = dzaz

C 2

= Z 2+ r

2

R

El mØtodo que usaremos serÆ calcular el potencial magnØtico vector y despuØs hallar el rotacional del mismo para calcular

B

.

Como el problema no nos dice nada, supondremos que el triÆngulo se encuentra en el vac o.

Calculamos:

  µ0 I a 2 dz az 2 2 µ0 I a z ln Z Z +r A= ∫ ⋅ + −a 2 2 2 = 4π 4π Z +r

(

a  +  I  2 A µ0 ln =4π ⋅ − a 2 + 

Ya podemos hallar

)

a 2 −a 2

 a 2    + r2  2   a z 2 a  −    + r2   2  

B

 calculando el rotacional en coordenadas cil ndricas

B= ∇×A= −aφ⋅∂∂Arz =µ0I a aφ

Ahora hallamos

B

2 4πr a + r 2  en los tres lados, lo cual se 4nos queda

B = 3⋅B= 34 πr Ia⋅2a+ r2 aφ TOTAL

4 Con esto, calculamos la intensidad magnØtica del conjunto

H = BTOTAL = 34 I⋅2a µ0

aφ

πr a4 + r2

Para simplificar el resultado, usaremos el teorema de PitÆgoras

a2 =

a2 h2 + 4

h= 3

(h − r )

2

a 2

2

2 3 a r  r2 a  2− = +4  

2

a 2 =r + 4

a

a

3 2 − 3ar+r2 = r2 + 42 4

a

r=

2 3

De tal forma, volviendo a la formula que estÆbamos desarrollando de la intensidad magnØtica llegamos a que:

H=

3I2

2

aφ 

a2 +2aπ32 ⋅= a44+a212a= a  SUSTITUYENDO→H= 2π33⋅I a3 aφ 4

12

12

3

En definitiva, el resultado final queda

 H = 9I aφ 2πa

Am−1

Problema 8

Ley de Biot-Savart

= µ4πo I ∫c dl ’R×2aR B ’

R ≡ distancia del punto punto

fuente

al

campo 

a R ≡ vector unitario del punto fuente al

punto campo

El circuito se divide en tres tramos: a) Corriente rectil nea sobre el eje z positivo b) Corriente rectil nea sobre el eje y negativo c) Corriente sobre la circunferencia de radio a a) Corriente rectil nea sobre el eje z positivo:

b) Corriente rectil nea sobre el eje y negativo: b

cosα=

⇒R= R

senα=

b

cosα

−y ⇒ y =−Rsenα=−b

=−btagα

cosα

R ⇒ dy =−b

senα

cosdα2α

 





dl’ x a R= -dy ⋅ 1 ⋅ senθ a X = -dycosα a µ

I

π

X

Bb =

    ’ d l a d l ’ x 2 aαRd=α0=  ⇒ R 2α cosb 4πoaX C∫ cosR 2αdy = µ4πo I aX ∫02 cosb  2 ’ cos α

Ba = 0

π

= µ4πobI aX ∫02cosαdα=

µ I Bb =

µ4πobI aX senα]π0 2 = µ4πobI aX

a 4πb X o

c) Corriente sobre la circunferencia de radio a:

  dl ’⊥aR 2

2

R = a +b cosθ = cosθ =

2

Z

a

=

⇒ dB Z = dB cosθ a 1

dB dB R

(a 2 + b 2 ) 2

Por simetr a, las componentes dBX se cancelan con los elementos dl situados en la parte opuesta del circuito.

o

dl’ xaR µo I

dl

µI

dB = 4π R 2

= 4π a 2 +b 2

θ BZ +

µ Ia

BZ +Iab 22 ) 32

o

2(a

µ Ia 2

 Bc = −

o

3

2(a + b ) 2





2

aZ 2



µI



2



BT = Ba + Bb + Bc = o 1 aX − (a22+πab2)32 aZ  π

4 b

µ Ia

µ2

Problema 9 Z

plano infinito Y

X

En primer lugar, como nos piden que estudiemos la densidad de flujo magnØtico para ambos lados del plano, distinguimos dichas regiones: (a) Para Y>0 (b) Para Y0 : Z

 aR ds’

 r’

  −r ’

R= r

 r

P

Y

X

Realizando los cambios:



−r

r

R

R = r ’

−r’

a= r−r’

y

Entonces, podremos expresar la ecuaci n anterior de la forma:

= µo ⋅∫S j ×(−rr−’ 3r



’)⋅ds’ (**) 4π ’ r

B Observando el dibujo, se

r





= y⋅a

r

y





tiene que: 



− r ’= y ⋅a y −



x’a x − z’a

z

r’= x’ax + z’az r−

r’ = y 2 + x’2 +z’2

Producto_vectorial ax

ay az

j ×(r



− r ’) = K 0 ⋅a z ×(y ⋅a y − x’a x −













z’a

K 0= −K 0 x’a y − K 0 ya x = K 0 (− ya x − x’a 





 y

z

)=0

)

− x’

y

− z’

0

Como existe simetr a respecto al eje Z, a la hora de integrar en la expresi n, los tØrminos en x positivos se van a cancelar con los tØrminos en x negativos, y por tanto, va a desaparecer la componente del eje Y. A

continuaci n, sustituyendo todos los datos en la expresi n (**), se obtiene:

µK(



o +∞+∞ 0

B

− ya

)⋅dx’⋅dz’

x

Como existe

simetr a respecto al eje Z, podremos tomar los l mites de la variable x en el intervalo [0,+∞) , y multiplicar por 2:

2⋅µ K ( B = o ⋅+ + ∫



∞−∞∞0

− yax)⋅dx’⋅3dz’ = −µo2⋅πK 0 ⋅ y ⋅+∫



0∞

+−∞∞(y 2 +

xdz’2 +’ z’2 )32 ⋅dx’⋅ax = 4π 0

(y

2

+ x’2 +z’2 )

2

= −µo2⋅πK 0 ⋅ y ⋅+∫∞( ’22 )( 2z’

’2

’2 )12 −+∞∞dx’⋅ax

= −µoπ⋅K 0 ⋅ y ⋅+∫0∞(xdx’2 +’⋅ayx2 ) = 0

 x +y

y + x +z



= −µoπ⋅K 0 ⋅ y  1y arctg xy’ ( T)

1

B=− µ0K0ax 2

 0+∞ax

Para ylb), número de espiras Na y Nb respectivamente, y considerando ambos radios iguales con valor R.

7) Una línea de transmisión coaxial, llena de aire, tiene un conductor interior sólido de radio a, y un conductor externo superficial de radio I b. Calcular la energía y la inductancia por unidad de longitud de la línea.

I a

b

indicado en la figura, circula una corriente cuya 

densidad es j = jor2a z . Una espira cuadrada, de lado a, está situada a una distancia 5a del eje de la corriente. El medio de permitividad µ1 ocupa el espacio de z>0, y el de permitividad µ2 el espacio de z 

I

=



=



. Con lo que,

Stapa

J ro 22π

rdr

donde se ha tenido en cuenta que en cil ndricas, ds =2πrdr

Ahora bien, debemos distinguir dos zonas: a) Fuera del cilindro. J πa3 Iext

J ro

rdr

b) Dentro del cilindro. J oπr3 Iint

J ro rdr

2

Cálculo del coeficiente de inducción mutua:

L =φ/ Iext donde φ es el flujo magnØtico que atraviesa la espira, debido a Iext:



s espira,

Bext ds

Para el cÆlculo de B utilizamos ley de ampere generalizada, teniendo en cuenta que el medio no es homogØneo. AdemÆs, por la simetr a del problema, la intensidad del campo magnØtico es tangente a la superficie de separaci n entre los dos materiales que conforman el medio. Con lo que, H1z -H2z = Js = 0 => H1z = H2z H1z = H1 H2z = H2 Por tanto, H1 = H2 = H La intensidad del campo magnØtico no va a depender del material en que se encuentre, tan s lo de la distancia a la corriente. Aplicando entonces la ley de Ampere generalizada a una circunferencia concØntrica con el eje del conductor, y de radio r: 



C

H *dl = 2πrH =I

Con lo que,

Hint = 2Iπintr aφ

Hext =



Iext a

φ

2πr La densidad de flujo magnØtico en la espira es distinta en cada material. As , con Hout, B1 = µ1Hext para z > 0 B2 = µ2Hext para z < 0

Y el flujo magnØtico a travØs de la espira vendrÆ dado por:

φ = ∫s1µ1H dsext  + ∫s2µ2H dsext  donde S ≡ S1+S2 es:

y ds = (a/2)•dr. Como la intensidad del campo magnØtico es perpendicular a la superficie rectangular que rodea la espira, es paralela a ds, con lo que Hext ds⇒Hext ⋅ds= Hext ⋅ds

y el flujo magnØtico viene dado por:

φ= dr +

Hext dr

Sustituyendo obtenemos: φ = [ J o a5ln(6/5) (µ1+µ2) ] / 8 Luego: L= φ /Iext = [a•(µ1+µ2) ln(6/5)]/4π Problema 9 a

I1 b

I2

2a

Tenemos que calcular la fuerza resultante sobre el circuito de la figura. Para ello lo primero que tenemos que saber es el campo magnØtico producido por el hilo rectil neo. De la teor a sabemos que el campo producido por un hilo rectil neo infinito es: B= µ0I aφ 2πR

siendo la corriente I, I1 en nuestro caso, y R la distancia al hilo infinito. Teniendo la expresi n del campo tenemos que buscar la expresi n de la fuerza

donde I es en esta expresi n la corriente Ique ejerce un conductor sobre otro y viene dada

por: Fm = I∫2d de la espira, l×B Bel campo magnØtico que existe a travØs de la espira, debido a I1, dl es el desplazamiento infinitesimal a lo largo de la espira y con el sentido de la corriente I2 . Ahora vamos a calcular las fuerzas que se producen en cada lado del circuito:

I2

4

a

b

B

Todos tienen misma direcci n y sentido indicados por 

1

Todas las flechas negras que no sean B

3 I1

 Todas las flechas grises

2

Fm

2a

dl

Aplicando sobre cada lado la expresi n de la fuerza, obtenemos la fuerza magnØtica total resultante. Ahora nos fijamos en la fuerza 2 y 4. Como vemos en el dibujo tienen la misma direcci n pero sentido contrario con lo que se restan. Pero si nos fijamos mÆs aœn vemos que el m dulo es tambiØn el mismo pues B var a de igual forma en un lado que en otro, debido a que la distancia a hilo recto infinito es la misma. As , las fuerzas magnØticas 2 y 4 se anulan. 

Fm4 =−Fm2



Nos quedan entonces las

F m1



y

F m3

, cuyas direcciones son las mismas pero sus

sentidos son contrarios, al igual que pasa con las fuerzas anteriores. Sin embargo, esta

vez los m dulos no van a ser iguales debido a los lados 1 y 3 no se encuentran a la misma distancia del hilo recto infinito. Es obvio que el modulo de F 



F



m1

va a ser mayor

B

que m3 , debido a que el campo es mÆs fuerte cuanto mÆs cerca del hilo infinito. Hallamos dichas fuerzas y las restamos para obtener la fuerza resultante.

Fm1 = I2∫dl×B= −I2∫dlBay =− I2B∫dlay =− I2Blay = −I2Bbay = −µ0I1I2 2πba ay  Fm = Fm1 + Fm3 = −µ0I1I2 2πba ay +µ0I1I2 ba − 12 + 14ay





4πba ay =µ0I1I2π

 Nota: Utilizamos coordenadas cartesianas para indicar la direcci n y sentido de las fuerzas. z y x Problema 10 (a) La densidad de flujo magnØtico sobre cada uno de los conductores: CONDUCTOR 1 I1

I2

I3

y

B21

B31

Como se trata de un hilo infinito, mediante la Ley de AmpØre:

µ

Bdl = 0I LlegÆbamos a la expresi n:

B= 2µπ0Ir aφ Para un hilo conductor rectil neo por el que circula una corriente I. En nuestro caso, la densidad de flujo en el conductor 1 serÆ la suma de la densidad de flujo del conductor 2 sobre el 1 y la del conductor 3 sobre el 1. As que:

B1 = B21 +B31  Por tanto, calcularemos B21 mediante la expresión antes citada: µ0

I B21 =2πd 2

Para darle carÆcter vectorial a la densidad de flujo nos vamos al dibujo y nos fijamos en el sentido de la corriente. Por la regla de la mano derecha, vemos que la densidad de flujo es perpendicular al plano yz, y con sentido hacia fuera. Por tanto, tendrÆ el sentido del eje positivo de las x :

B21 =µ2π0 2Id ax Calculamos a continuaci n la densidad de flujo del conductor 3 sobre el 1:

B31 = 2 (2 )πµ µ0 3Id = 4π0 3Id ax

Si sumamos ambas expresiones y sacamos factor comœn:

4µπ0d[I3 +2I a2]X B1 =

Obtendremos la densidad de flujo magnØtico en el conductor 1. CONDUCTOR3

I1

I2

I3  B 23 

 B13 

 B13 

Por simetr a, para el conductor 3 obtendremos:

B23 =−µ2π0 2Id ax Si le damos carÆcter vectorial veremos que al ir la corriente hacia arriba, por la regla de la mano derecha, la densidad de flujo es perpendicular al plano yz y hacia adentro(en el sentido negativo del eje de las x). Para la densidad de 1 sobre 3 :

µ

B13 =−2 (2 )π



µ

Id a x =− 4π0 1Ida

01



x

Si sumamos ambas expresiones y sacamos factor comœn:



4µπ0d[I1+2I

a2]X B3 =−

Obtendremos la densidad de flujo para el conductor 3.

CONDUCTOR 2 I1

I2

I3

 B12  B  31 Calcularemos: B2 =

B12 + B32

Por tanto:

B12 =−µ2π0 1Idax Para darle carÆcter vectorial nos fijamos en el sentido de la corriente. Como va hacia arriba, por la regla de la mano derecha, la densidad de flujo serÆ perpendicular al plano yz y hacia adentro. Por tanto, en el sentido negativo del eje de las x. Para el caso de

B32 serÆ en el sentido positivo del eje de las x, ya que la densidad de flujo serÆ perpendicular al plano yx y hacia fuera. B32 =µ2π0 3Id

ax

Sumamos y sacamos factor comœn:

2µπ0d[I3 −I a1]X B2 = (b) La fuerza por unidad de longitud que aparece sobre cada uno de los conductores. CONDUCTOR 1

I1

I2

I3

dl  F 21   F 21 

La fuerza en el conductor 1 serÆ la suma de la fuerza que ejerce el conductor 2 sobre el

  F y la que ejerce el 3 sobre el 1. 1 =F21+ F31

1

La calcularemos mediante la expresi n de la fuerza magnØtica en un conductor: F



B×

El

dl

=I∫Cdl

tendrÆ la direcci n y sentido de la corriente. Calculamos F21 :

F21 = I1∫Cdl B⋅ 21 = µ20 1 2π d ∫Cdz II

 Donde hemos tenido en cuenta que el diferencial de longitud ( perpendicular a



y ademÆs que dl = dz.

dl ) es

B

Por tanto, tenemos que: Fl21 µ20 1 2πI

Id ay

= Para darle carÆcter vectorial tenemos en cuenta que se trata de dos corrientes paralelas en el mismo sentido, con lo que la fuerza tendrÆ el sentido positivo del eje de las y.

La contribuci n del conductor 3 serÆ: Fl31 µ40 1 3πI

Id ay

=

Si sumamos y sacamos factor comœn: Fl1 =

µ4π0 1Id [I3 + 2I2]ay

Obtendremos la fuerza por unidad de longitud para el conductor 1. CONDUCTOR 3 I1 I2

I3  dl   F 13   F 23

Para el caso del conductor 3, por analog a con el conductor 1, calcularemos la fuerza por la expresi n del caso anterior, para la fuerza por unidad de longitud del conductor 2 sobre el 3 obtendremos: Fl23 =

−µ20 2 3πI Id ay

Para darle carÆcter vectorial, como en el caso anterior, teniendo en cuenta la regla de la mano derecha y teniendo en cuenta que se trata de dos corrientes paralelas del mismo sentido y que, ademÆs los conductores 1 y 2 estÆn situados a la izquierda del conductor 3, obtendremos que la fuerza irÆ en el sentido negativo del eje de las y, como podemos observar el la figura.

Para la contribuci n del conductor 1 obtendremos: Fl13 =

Si sumamos y sacamos factor comœn: Fl3 =

−µ40 1 3πI Id ay

−µ4π0 3Id [I1 + 2I2]ay

Obtendremos la fuerza por unidad de longitud para el conductor 3.

CONDUCTOR 2 I1 I2  dl  

 F 12 

I3

 F 32 

Para el caso del conductor 2, utilizaremos las mismas expresiones que en los casos anteriores. Para la fuerza por unidad de longitud del conductor 3 sobre el 2, obtendremos: Fl32 µ0 2 3πI

Id ay

=2 Para darle carÆcter vectorial tendremos en cuenta la regla de la mano derecha y que el conductor 3 estÆ situado a la derecha del conductor 2, donde queremos calcular la fuerza. Obtendremos que la fuerza irÆ en el sentido positivo del eje de las y, como podemos observar el la figura.

Para la contribuci n del conductor 1 obtendremos: Fl12 =

−µ20 1 2πI Id ay

Para darle carÆcter vectorial tendremos en cuenta la regla de la mano derecha y que el conductor 1 estÆ situado a la izquierda del conductor 2, donde queremos calcular la fuerza. Obtendremos que la fuerza irÆ en el sentido negativo del eje de las y, como podemos observar el la figura.

Si sumamos y sacamos factor comœn: Fl2 µ2π0 2Id

[I3 − I a1]y

= Obtendremos la fuerza por unidad de longitud para el conductor 2 Problema 11

I

I

I

 B Mercurio a)

b)

c)

Como sabemos, cuando tenemos un conductor por el que circula una corriente I en una regi n del espacio donde existe un campo magnØtico, sobre dicho conductor aparecerÆ una fuerza magnØtica que viene determinada por:

Nos piden que averig emos quØ espira gira, por lo que tendremos que realizar un anÆlisis del sentido de las fuerzas en cada sector del circuito, y luego interpretar si Østas se van a compensar, o van a hacer que la espira gire.

a) I en

6

5

No

En el dibujo podemos observar la direcci n y sentido de los dl rojo).



(vectores

ta(1): tener en cuenta que si queremos colocar En todos ellos se va a 8

cumplir que y como Comencemos nuestro estudio por los

7

conductores verticales:

B

dl= dz⋅az (Ver Notadlen forma m =0; (1)) y dl son paralelos, dl×B=0, por lo que F

de m dulo-vector, dicho m dulo es dz, porque un elemento diferencial es un incremento infinitesimal, as que como

z vamos en el sentido decreciente de las z entonces dz por si solo serÆ negativo. Por otro lado su carÆcter vectorial serÆ -, por lo que simplificando quedarÆ  siempre va en el mismo sentido de la corriente. dl= dz⋅az.

az

Nota(2): dl y

x

Continuamos por el conductor 5: = dy ⋅ay y dl × B=

dy⋅ B0 ⋅ax En Øl dl Llevando este resultado a la integral obtenemos que:

L   IB⋅ I Fm = ∫02 dy ⋅B ax 0 ⋅ = 2

0⋅

 ⋅ La⋅ x N 4

Tener en cuenta que la corriente que pasa por el conductor es I/2, ya que suponiendo conductores hmicos y como ambas ramas tienen la misma longitud, entonces la resistencia que provocaran al paso de corriente es la misma. Conductor 6: Nuevamente el proceso es el mismo: dl= dy ⋅ay (ver nota (1)) 

dl× B= −dy ⋅ Bo⋅(−ax ) = dy ⋅ Bo⋅ax



Por lo que: 



I

Fm = ∫−L2 dy ⋅ Bo⋅ax= − I ⋅ Bo⋅ L⋅ax N 0 2

4

Conductor 7:= dy ⋅ay (ver nota (1)) dl dl× B= −dy ⋅ 



Bo⋅(−ax ) = dy ⋅ Bo⋅ax la f rmula:

Sustituyendo en

   I 0 I ⋅ Bo ⋅ L ⋅ ax Fm = ∫ L dy Bo ax N 2 2 ⋅ ⋅ =− 4

Conductor 8:= dy ⋅ay dl dl× B= dy ⋅ 

Bo⋅ax

Por tanto:    I ⋅ Bo ⋅ L ⋅ ax I 0 Fm = ∫L dy ⋅ Bo ⋅ ax = N 2 −2 4

GIRA. Como se puede observar tanto las fuerzas sobre los conductores 5 y 7 como las de 6 y 8 se van a compensar porque tienen el mismo m dulo y sentido contrario. Por tanto esta espira NO

Notar que las fuerzas sobre cada conductor son constantes debido a que el campo es fijo. El diagrama de fuerzas quedar a de la siguiente manera:

b) El procedimiento va a ser totalmente anÆlogo:

5

I

En primer lugar hay que tener en cuenta la corriente I . Suponiendo conductores hmicos la resistividad que provocan al paso de la corriente es: R=σ

I

L⋅ S

donde L es la longitud, σ es la conectividad y S es la secci n. Tenemos dos ramas, una de longitud L y otra de 3L, entonces como la conductividad y la

6

secci n es la misma, la resistencia de cada conductor serÆ R y 3R. Por tanto, haciendo un divisor de corriente, obtenemos que I’= I de donde se deduce 4 que por el otro conductor irÆ I’’=

3I

.

4

Nuevamente en los conductores verticales



 y B son paralelos:

el producto vectorial va

a ser cero, ya que dl dl×B=0

Y entonces:

F



m =0

Conductor  5:  dl = dy ⋅ ay  

dl × B = dy⋅ B0 ⋅ax Por tanto:



I B L a xN dy B ax

Conductor 6:dl= dy

⋅ay (

ver nota (1))

   dl × B = −dy Bo⋅(−ax ) = dy ⋅ Bo⋅ax ⋅ Por lo que:

I Bo L a⋅ ⋅

⋅ x

N

dy Bo ax

Como se observa, las fuerzas sobre el conductor 5 y el 6 se compensan, ya que son de sentido contrario, por lo que la espira NO GIRA. El diagrama de fuerzas ser a:

c)

6

5

El estudio de esta espira se puede realizar a partir de los resultados obtenidos en a). En los conductores verticales no van a aparecer fuerzas. Las expresiones de las fuerzas que aparecen sobre 5 y 6 son exactamente las mismas que en 5 y 6 de a):

Conductor 5: L  I Fm = ∫02 dy ⋅B 2

0

 IB ⋅ ⋅ ax =

0⋅

 ⋅ L ⋅a x N 4

Conductor 6: L   I Bo I −2 ⋅ L⋅a ⋅  xN Fm = ∫ dy Bo ax ⋅ ⋅ =− 2 0 4

Tener en cuenta que el mercurio es un conductor l quido, es decir va a permitir que la corriente circule por el y a la vez no va a influenciar sobre el movimiento de la espira. El diagrama de fuerzas ser a:

Como se puede observar esta espira SÍ GIRA. Problema 12

Resolveremos el problema sabiendo que el sistema es un circuito magnØtico, para ello primero calcularemos las reluctancias de ambos materiales que llamaremos R1 y R2 para el material 1 y 2 respectivamente. La reluctancia viene dada por la longitud del material partida del producto de su permeabilidad y su secci n (para el caso de un toroide), por tanto: R1 =

µba2

R2 =

µ2ba2

1

Como ambas reluctancias estÆn colocadas en serie (el flujo que las atraviesa es el mismo, esto es as ya que b>>a y a las condiciones de continuidad), la reluctancia total del circuito serÆ la suma de ambas reluctancias y por tanto el flujo por el circuito serÆ: Φ = NIµ1µ2a2 (µ1 +µ2 )b Por otro lado como el flujo a travØs del toroide es constante tambiØn la densidad de flujo magnØtico lo serÆ, y ademÆs al ser la secci n del toroide constante, tenemos que: Φ = ∫ B→•ds→ = ∫ B•ds = B∫ds = Bπa2 Y finalmente tenemos:

B→2 =

NIµ B→1 = 1µ2 →aϕ (µ1 +µ2 )πb

H→ 1 = B→1/µ1 = (µ1NI+µµ22)πb →aϕ

H→ 2 = B→1/µ2 = (µ1 NI+µµ21)πb →aϕ

ELECTRICIDAD Y MAGNETISMO. Electromagnetismo 1) Calcular la fuerza electromotriz inducida en una espira por un par de hilos paralelos de gran longitud, por los que circula una corriente igual pero con sentidos contrarios. a I

I h b



2)

En un semiespacio z > 0 existe un campo magnético, B= B·a x , constante. Un circuito plano, contenido

en el plano x= 0, está formado por una semicircunferencia de radio R y centro O, limitada por un diámetro, construidos con un material conductor homogéneo, cuya resistencia por unidad de longitud es ρ=

π



. Este

(rad/s), estando inicialmente

circuito gira en su plano alrededor de O con velocidad angular contante, ω= 2·ax el diámetro coincidiendo con el eje X, y la semicircunferencia en la región z > 0. Calcular:



a)

El flujo del campo magnético B= B·a x a través del circuito. Representar gráficamente su valor en función del tiempo.

b) La fuerza electromotriz inducida en el circuito. c) La intensidad de la corriente eléctrica que circula por el circuito, indicando su sentido, en función del tiempo. Representarla gráficamente, tomando como sentido positivo el contrario al de las agujas del reloj. 

d) La fuerza que el campo magnético B= B·a x ejerce sobre el circuito.

Problema 1 Calcular la fuerza electromotriz inducida en una espira por un par de hilos paralelos de gran longitud, por los que circula una corriente igual pero con sentidos contrarios. 2

1 ε

a

Z

dS I r2 h r1 I b

Y X

l

ε= − ddtφ ; B2•dS2

=

B1 µ2π0rI1 (−

B2 = 2µπ0rI2 (

φ1 = ∫ B1•dS1 , φ2 = ∫

φ=φ1 +φ2 ;

a x

a

x

)

(Deducimos las direcciones y sentidos de los campos

)

atendiendo a los sentidos de las corrientes)

φ1 = a∫+l −2πµr0I (− h•dr1) = µ20π•Ir1•h Ln aa+ l ; φ2 = b∫b+l µ2π0rI1

(− h•dr2) = −µ2π0r•1I•h Lnbb+ l 

d

φ= µ0Ih Ln ba((ab ++ ll)) 2π

ε= −



dt

ε= −µ0h Lnba((ab ++ ll))

dIdt

2π Problema 2

α

a) ω=π (rad /s) , y también ω= 2

,

ds t

α

siendo αel Ængulo recorrido en funci n del tiempo t como se indica en el dibujo :

φLa formula del flujo es: = ∫s •ds= ∫sB•ds = B∫sds = B•S siendo S la B superficie del circuito atravesada por el campo. Se considera que B es paralelo a ds, por lo que su producto escalar es igual al producto de sus m dulos. AdemÆs, como B tiene modulo constante lo podemos sacar de la integral. La superficie S en cada instante serÆ, aplicando una regla de 3 con una circunferencia (Ængulo 2π)

π−α 2 , entonces S=

sustituyendo: (Wb)

•R α

2 De la formula anterior de ω= π•t

ω, α=

, obtenemos α=ω•t , y sustituyendo

t 2

Entonces φ= •BR2, con t ∈[0,2] (estos valores de t son para completar media vuelta, es decir, que la espira salga del campo.)

A partir de t = 0, instante en el que la espira se encuentra completamente dentro del campo, el desplazamiento la va sacando de Øl, por lo que la expresi n del flujo es decreciente hasta t = 2, instante en el que la espira estÆ completamente fuera del campo (y el flujo se anula). Para t ∈[2,4] se obtiene una expresi n similar, pero el flujo es ahora creciente, dado que la espira va entrando en el campo. En adelante la grafica serÆ peridica.

φ π •B•R 2

2

2

t (s)

4

b) La expresi n de la fuerza electromotriz inducida viene dada por la variaci n del flujo respecto al tiempo: dφ π − = •BR dt

2

(V) ξ= 4

ξ π •B•R 2 4 t (s)

π •B•R 2 − 4

2

4

6

c) La corriente viene dada por la ley de Ohm, que dice que la intensidad es la relaci n entre la tensi n y la resistencia del conductor, siendo la tensi n la f.e.m: , y la resistencia es ℜ=ρ•(πR+

I=

2R) =R , siendo (πR + 2R)la longitud de

la espira (πR es la longitud de la semicircunferencia y 2R el diÆmetro) Con lo que queda:

π•BR2 4 I =•BR1 (A) 2 +π



•(πR + 2R) I

π •B•R 4

4

ξ

t (s)

π •B•R − 4

2

4

6

d) Dividimos el camino de la espira en 2 partes, a la semicircunferencia la llamaremos C1, y al diÆmetro lo llamaremos C2. La formula de la fuerza magnØtica es: dl

dlxB B

F= I•C∫ dlxB

= F 1 + F 2 = I•∫C1dlxB





 

dlxB

C2

En C1 , Cuando 0